Alan Anderson 30 de janeiro de 2017 · 6.4.10 Áreas para encontrar razões entre ... façam o...

88

Transcript of Alan Anderson 30 de janeiro de 2017 · 6.4.10 Áreas para encontrar razões entre ... façam o...

Page 1: Alan Anderson 30 de janeiro de 2017 · 6.4.10 Áreas para encontrar razões entre ... façam o mesmo com os outros níveis e ... Agora esse texto visa ajudar estudantes que não sabem

OBM - Segundo Contato

Alan Anderson

30 de janeiro de 2017

Page 2: Alan Anderson 30 de janeiro de 2017 · 6.4.10 Áreas para encontrar razões entre ... façam o mesmo com os outros níveis e ... Agora esse texto visa ajudar estudantes que não sabem

2

Page 3: Alan Anderson 30 de janeiro de 2017 · 6.4.10 Áreas para encontrar razões entre ... façam o mesmo com os outros níveis e ... Agora esse texto visa ajudar estudantes que não sabem

Sumário

0.1 Pra que esse livro? . . . . . . . . . . . . . . . . . . . . . . . . 60.2 Pra quem é esse texto? . . . . . . . . . . . . . . . . . . . . . . 70.3 Pré-requisitos . . . . . . . . . . . . . . . . . . . . . . . . . . . 7

1 Problemas de Lógica e Teoria dos conjuntos 9

2 Combinatória e Probabilidade 132.1 Exercícios de Contagem . . . . . . . . . . . . . . . . . . . . . 132.2 Números binomiais . . . . . . . . . . . . . . . . . . . . . . . . 15

2.2.1 Fatorial . . . . . . . . . . . . . . . . . . . . . . . . . . 152.2.2 Número Binomial . . . . . . . . . . . . . . . . . . . . . 152.2.3 Justi�cando o nome . . . . . . . . . . . . . . . . . . . . 16

2.3 Princípio das casas dos pombos . . . . . . . . . . . . . . . . . 172.4 Jogos e Invariantes(Seção de exemplos) . . . . . . . . . . . . . 182.5 Probabilidade . . . . . . . . . . . . . . . . . . . . . . . . . . . 19

2.5.1 Probabilidade Discreta . . . . . . . . . . . . . . . . . . 202.5.2 Probabilidade Geométrica . . . . . . . . . . . . . . . . 212.5.3 Exercícios da OBM . . . . . . . . . . . . . . . . . . . . 22

3 Álgebra 253.1 Produtos Notáveis e Fatorações Notáveis . . . . . . . . . . . . 25

3.1.1 Lista de produtos notáveis . . . . . . . . . . . . . . . . 253.1.2 Completando Quadrados . . . . . . . . . . . . . . . . . 26

3.2 Binômio de Newton . . . . . . . . . . . . . . . . . . . . . . . . 273.3 Equação do Segundo Grau . . . . . . . . . . . . . . . . . . . . 273.4 Polinômios . . . . . . . . . . . . . . . . . . . . . . . . . . . . . 303.5 Desigualdes . . . . . . . . . . . . . . . . . . . . . . . . . . . . 31

3.5.1 Exercícios de aquecimento . . . . . . . . . . . . . . . . 313.5.2 MA ≥ MG . . . . . . . . . . . . . . . . . . . . . . . . 32

3

Page 4: Alan Anderson 30 de janeiro de 2017 · 6.4.10 Áreas para encontrar razões entre ... façam o mesmo com os outros níveis e ... Agora esse texto visa ajudar estudantes que não sabem

4 SUMÁRIO

3.6 Função Parte Inteira . . . . . . . . . . . . . . . . . . . . . . . 333.7 Recorrências . . . . . . . . . . . . . . . . . . . . . . . . . . . . 34

3.7.1 Exercícios da OBM . . . . . . . . . . . . . . . . . . . . 343.7.2 PA e PG . . . . . . . . . . . . . . . . . . . . . . . . . . 353.7.3 Equação Característica (escrever) . . . . . . . . . . . . 363.7.4 Pequenas modi�cações resolvem problemas (seção de

exemplos) . . . . . . . . . . . . . . . . . . . . . . . . . 363.8 Equações e sistemas(Seção de exemplos) . . . . . . . . . . . . 373.9 Equações funcionais(Seção de exemplos) . . . . . . . . . . . . 40

3.9.1 Exercícios com funções . . . . . . . . . . . . . . . . . . 40

4 Teoria dos Números 434.1 Divisibilidade . . . . . . . . . . . . . . . . . . . . . . . . . . . 434.2 Teorema Fundamental da Aritmética . . . . . . . . . . . . . . 454.3 Congruências . . . . . . . . . . . . . . . . . . . . . . . . . . . 464.4 Exercícios da OBM primeira fase . . . . . . . . . . . . . . . . 474.5 Pequeno Fermat e Grande Euler . . . . . . . . . . . . . . . . . 494.6 Teorema Chinês dos Restos . . . . . . . . . . . . . . . . . . . 50

5 Geometria 515.1 Fatos básicos . . . . . . . . . . . . . . . . . . . . . . . . . . . 515.2 Pitágoras e Triângulo Retângulo . . . . . . . . . . . . . . . . . 525.3 Informações básicas de trigonometria . . . . . . . . . . . . . . 54

5.3.1 Seno e Cosseno como funções reais . . . . . . . . . . . 545.3.2 Ângulos notáveis . . . . . . . . . . . . . . . . . . . . . 555.3.3 Fórmulas úteis . . . . . . . . . . . . . . . . . . . . . . 56

5.4 Lei dos Cossenos . . . . . . . . . . . . . . . . . . . . . . . . . 575.5 Ângulos Inscritos e Ângulos Tangentes . . . . . . . . . . . . . 57

5.5.1 Ângulos inscritos . . . . . . . . . . . . . . . . . . . . . 575.5.2 Ângulos Tangentes . . . . . . . . . . . . . . . . . . . . 58

5.6 Pontos, retas e segmentos notáveis . . . . . . . . . . . . . . . . 585.6.1 Bissetrizes . . . . . . . . . . . . . . . . . . . . . . . . . 585.6.2 Medianas . . . . . . . . . . . . . . . . . . . . . . . . . 595.6.3 Alturas . . . . . . . . . . . . . . . . . . . . . . . . . . 595.6.4 Mediatrizes . . . . . . . . . . . . . . . . . . . . . . . . 60

5.7 Áreas de �guras planas . . . . . . . . . . . . . . . . . . . . . . 605.7.1 Área do triângulo . . . . . . . . . . . . . . . . . . . . . 61

5.8 Quadriláteros . . . . . . . . . . . . . . . . . . . . . . . . . . . 62

Page 5: Alan Anderson 30 de janeiro de 2017 · 6.4.10 Áreas para encontrar razões entre ... façam o mesmo com os outros níveis e ... Agora esse texto visa ajudar estudantes que não sabem

SUMÁRIO 5

6 Enunciados dos Problemas 636.1 Combinatória e Probabilidade . . . . . . . . . . . . . . . . . . 63

6.1.1 classi�car . . . . . . . . . . . . . . . . . . . . . . . . . 636.1.2 Contagem . . . . . . . . . . . . . . . . . . . . . . . . . 646.1.3 Jogos e Invariantes . . . . . . . . . . . . . . . . . . . . 676.1.4 Probabilidade . . . . . . . . . . . . . . . . . . . . . . . 69

6.2 Álgebra . . . . . . . . . . . . . . . . . . . . . . . . . . . . . . 706.2.1 Funcões e Recorrências . . . . . . . . . . . . . . . . . . 706.2.2 Somatórios e Produtórios . . . . . . . . . . . . . . . . . 726.2.3 Equações do segundo grau e Polinômios . . . . . . . . 726.2.4 Parte Inteira e Aproximação . . . . . . . . . . . . . . . 736.2.5 Cotas e desigualdades . . . . . . . . . . . . . . . . . . 746.2.6 Várias variáveis . . . . . . . . . . . . . . . . . . . . . . 74

6.3 Teoria dos Números . . . . . . . . . . . . . . . . . . . . . . . . 756.3.1 Equações e divisibilidade . . . . . . . . . . . . . . . . . 756.3.2 Teorema Fundamental da Aritimética . . . . . . . . . . 776.3.3 Teorema Chinês dos Restos e Sistemas de Congruências 786.3.4 Fermat, Euler e Wilson . . . . . . . . . . . . . . . . . . 78

6.4 Geometria . . . . . . . . . . . . . . . . . . . . . . . . . . . . . 796.4.1 Classi�car . . . . . . . . . . . . . . . . . . . . . . . . . 796.4.2 A soma dos ângulos internos . . . . . . . . . . . . . . . 826.4.3 Semelhança de Triângulos e Relações Métricas em Tri-

ângulos Retângulos . . . . . . . . . . . . . . . . . . . . 826.4.4 Lei dos Cossenos . . . . . . . . . . . . . . . . . . . . . 836.4.5 Lei dos senos . . . . . . . . . . . . . . . . . . . . . . . 836.4.6 Desigualdade Triangular . . . . . . . . . . . . . . . . . 836.4.7 Área do Triângulo . . . . . . . . . . . . . . . . . . . . 836.4.8 Pontos notáveis . . . . . . . . . . . . . . . . . . . . . . 846.4.9 Potência de Ponto . . . . . . . . . . . . . . . . . . . . 846.4.10 Áreas para encontrar razões entre segmentos . . . . . . 84

7 Ideias e Soluções 877.1 Soluções Álgebra . . . . . . . . . . . . . . . . . . . . . . . . . 87

7.1.1 Sequências e funções . . . . . . . . . . . . . . . . . . . 87

Page 6: Alan Anderson 30 de janeiro de 2017 · 6.4.10 Áreas para encontrar razões entre ... façam o mesmo com os outros níveis e ... Agora esse texto visa ajudar estudantes que não sabem

6 SUMÁRIO

0.1 Pra que esse livro?

Versão 3Espero que com esse material você possa ter uma referência de estudo para

a segunda fase da OBM. Algumas questões não foram classi�cadas em umtópico especí�co e caso você resolva, ou veja uma solução, e queira ajudar naclassi�cação, por favor mande um email para [email protected].

Um dos objetivos desse material também é inspirar que outras pessoasfaçam o mesmo com os outros níveis e as outras fases, caso não apareçaninguém querendo fazer, eu continuarei pacientemente.

Essa é uma versão intermediária do material, que inclui os problemasainda não completamente classi�cados e com problema com respeito as �gu-ras. Esses pequenos problemas serão resolvidos em versões posteriores. Casovocê encontre algum erro de digitação, ou algo copiado errado, por favor, meenvie um email (se você tentar resolver o problema e ele parecer impossívelou trivial, talvez seja bom ir na prova e ver se o enunciado está correto).

Se você tiver sugestões para melhorar as classi�cações, por favor, me envieum email. O objetivo é fazer classi�cações que usem conhecimentos básicose simples e também deixar os problemas bem distribuídos (o que fazer emcontagem?). Algumas vezes, alguns problemas poderão estar em mais deuma classe, e em geral quando isso acontecer, eles serão colocados na classeque tem menos problemas, para que haja uma distribuição mais equilibrada.É óbvio (pra quem?) que você pode resolver os problemas com técnicas quenada tenham a ver com a da classe, mas tenha em mente que o problemapode ser resolvido com aquela técnica e tente resolvê-lo usando-a, pois um dosobjetivos do material é que se aprenda as técnicas dos tópicos, caso contráriovocê poderia simplesmente ir pegando as provas da OBM aleatóriamente,sem precisar da divisão por assunto.

Em versões posteriores haverão também sugestões e soluções dos proble-mas. As sugestões serão baseadas no que eu usar para resolver os problemas,ou serão soluções para casos diferentes dos pedidos no enunciado. A soluçõesserão soluções diferentes das oferecidas pelo site da OBM, e se você quiserme enviar sua solução, seguirei o modelo da revista Eureka! pondo seu nomecompleto, cidade e estado. Por questão de volume de trabalho só irei porsoluções que me forem enviadas digitada no latex (o arquivo de edição - ctrlC ctrl V), porque senão será muito trabalho (já �co louco só pra digitar asminhas próprias).

Então é isso. Em caso de sugestão, correção (inclusive de ortogra�a ou

Page 7: Alan Anderson 30 de janeiro de 2017 · 6.4.10 Áreas para encontrar razões entre ... façam o mesmo com os outros níveis e ... Agora esse texto visa ajudar estudantes que não sabem

0.2. PRA QUEM É ESSE TEXTO? 7

concordância), agradecimento ou pedido de casamento, envie-me um email:[email protected].

0.2 Pra quem é esse texto?

Inicialmente esse texto seria somente os problemas da segunda fase separadospor área, e comecei a fazer isso para ajudar alguns amigos que tenho que naépoca estudavam no IF-AL. Mas trabalhos geram ideias que dão trabalho.Agora esse texto visa ajudar estudantes que não sabem exatamente comoestudar para as olimpíadas. Escrevi um texto que eu gostaria de ter lidoquando fazia ensino médio e estudava para as olimpíadas, espero que sejaútil.

Para aluno completamente iniciante, talvez o texto seja um pouco difí-cil, pois pressupõe que o leitor já saiba algumas coisas e que tenha certafamiliaridade com demonstrações. Se você acha que a prova da segunda faseé fácil, esse texto talvez não tenha muito a acrescentar na sua vida tam-bém. Eu classi�caria como um texto de nível intermediário, que pressupõesconhecimentos básicos e familiaridade com demonstrações.

Outra singularidade do texto é o fato de que ele está cheio de resultadossem demonstrações, e muitos deles com convite ao leitor a desenvolver ademonstração. As proposições em que o leitor é convidado a demonstrar,em geral são simples, e por simples quero dizer que é algo que talvez vocêache fácil, talvez ache difícil de fazer ao tentar, mas que quando você vera demonstração provavelmente pensará "ah, é só isso". Em geral, serãodeixadas nos esboços as idéias principais das provas, �cando para o leitor otrabalho de fazer as contas ou de usar idéias "óbvias"ou ideias anteriormenteapresentadas no próprio texto.

0.3 Pré-requisitos

É esperado que o leitor desse livro se familiarize com alguns temas antes decomeçar a lê-lo. As de�nições mais básicas e alguns resultados simples decada tema muitas vezes não são apresentadas, por algumas razões. Por essemotivo, indicamos referências iniciais para quem quer ler o conteúdo daqui.Esse livro é mais um guia de estudo do que um curso por si próprio.

Page 8: Alan Anderson 30 de janeiro de 2017 · 6.4.10 Áreas para encontrar razões entre ... façam o mesmo com os outros níveis e ... Agora esse texto visa ajudar estudantes que não sabem

8 SUMÁRIO

Em Combinatória esperamos que o leitor conheça os princípios aditivo emultiplicativo, esses são extremamente simples e são a base da combinatória.

Em Geometria é esperado que se saiba semelhança de triângulos, a de�-nição de seno e cosseno.

Em Teoria dos números, conhecer o conceito de congruência e saber aspropriedades aritméticas básicas.

Em Álgebra, é esperado que se saiba, mesmo que vagamente, o que é umafunção e um polinômio (e suas operações).

Uma possibilidade para se adquirir os pre-requisitos é ler as apostilas daOBMEP.

Page 9: Alan Anderson 30 de janeiro de 2017 · 6.4.10 Áreas para encontrar razões entre ... façam o mesmo com os outros níveis e ... Agora esse texto visa ajudar estudantes que não sabem

Capítulo 1

Problemas de Lógica e Teoria dosconjuntos

1998. Um crime é cometido por uma pessoa e há quatro suspeitos: André,Eduardo, Rafael e João. Interrogados, eles fazem as seguintes declarações:

- André: Eduardo é o culpado.- Eduardo: João é o culpado.- Rafael: Eu não sou culpado.- João: Eduardo mente quando diz que eu sou culpado.Sabendo que apenas um dos quatro disse a verdade, quem é o culpado?

1998. Se x homens fazem x embrulhos em x segundos, em quantos se-gundos y homens farão y embrulhos?

1998. A respeito da resposta de um problema, Maurício, Paulo, Eduardoe Carlos �zeram as seguintes a�rmações:

- Maurício: É maior que 5.- Paulo: É menor que 10.- Eduardo: É um número primo.- Carlos: É maior que 12.Entre as a�rmações acima, quantas, no máximo, podem ser verdadeiras?

1999. Em um hotel há 100 pessoas. 30 comem porco, 60 comem galinha e80 comem alface. Qual é o maior número possível de pessoas que não comemnenhum desses dois tipos de carne?

9

Page 10: Alan Anderson 30 de janeiro de 2017 · 6.4.10 Áreas para encontrar razões entre ... façam o mesmo com os outros níveis e ... Agora esse texto visa ajudar estudantes que não sabem

10CAPÍTULO 1. PROBLEMAS DE LÓGICA E TEORIA DOS CONJUNTOS

1999. Uma caixa contém 100 bolas de cores distintas. Destas, 30 sãovermelhas, 30 são verdes, 30 são azuis e entre as 10 restantes, algumas sãobrancas e outras são pretas. Qual é o menor número de bolas que devemostirar da caixa, sem lhes ver a cor, para termos a certeza de haver pelo menos10 bolas da mesma cor?

2000. Há três cartas viradas sobre uma mesa. Sabe-se que em cada umadelas está escrito um número inteiro positivo. São dadas a Carlos, Samuel eTomás as seguintes informações:

i) todos os números escritos nas cartas são diferentes;ii) a soma dos números é 13;iii) os números estão em ordem crescente, da esquerda para a direita.Primeiro, Carlos olha o número na carta da esquerda e diz: ?Não te-

nho informações su�cientes para determinar os outros dois números.? Emseguida, Tomás olha o número na carta da direita e diz: ?Não tenho informa-ções su�cientes para determinar os outros dois números.? Por �m, Samuelolha o número na carta do meio e diz: ?Não tenho informações su�cientespara determinar os outros dois números.? Sabendo que cada um deles sabeque os outros dois são inteligentes e escuta os comentários dos outros, qual éo número da carta do meio?

2000. (FIGURA) Juliano colou uma bandeirinha cinza em cada engrena-gem, como mostra a �gura abaixo:

As engrenagens são iguais e quando a engrenagem da esquerda girou umpouco, a sua bandeirinha �cou na posição indicada com a bandeirinha brancapontilhada. Nesta condição, podemos a�rmar que a posição da bandeirinhana engrenagem da direita é:

2001. Cinco animais A,B,C,D, e E, são cães ou são lobos. Cães semprecontam a verdade e lobos sempre mentem. A diz que B é um cão. B dizque C é um lobo. C diz que D é um lobo. D diz que B e E são animais deespécies diferentes. E diz que A é um cão. Quantos lobos há entre os cincoanimais?

2001. (FIGURA) Somente uma das �guras a seguir representa a plani�ca-ção de um cubo na qual está destacada a sua interseção com um plano. Qual?

Page 11: Alan Anderson 30 de janeiro de 2017 · 6.4.10 Áreas para encontrar razões entre ... façam o mesmo com os outros níveis e ... Agora esse texto visa ajudar estudantes que não sabem

11

2002. Qual é a quantidade total de letras de todas as respostas incorretasdesta questão?

A) Quarenta e oito.B) Quarenta e nove.C) Cinqüenta.D) Cinqüenta e um.E) Cinqüenta e quatro.

2003. A seqüência ”22” descreve a si mesma, pois ela é formada porexatamente dois 2. Analogamente, a seqüência ”31123315” descreve a simesma, pois é formada por exatamente três 1, um 2, três 3 e um 5. Qual dasseguintes seqüências não descreve a si mesma?

A) 21322316B) 31123318C) 3122331719D) 2132332415E) 41322324151618

2003. Você está em um país estrangeiro, a LUCIÂNIA, e não conheceo idioma, o LUCIANÊS, mas sabe que as palavras ?BAK? e ?KAB? signi-�cam sim e não, porém não sabe qual é qual. Você encontra uma pessoaque entende português e pergunta: "KAB signi�ca sim?"A pessoa responde"KAB". Pode-se deduzir que:

A) KAB signi�ca sim.B) KAB signi�ca não.C) A pessoa que respondeu mentiu.D) A pessoa que respondeu disse a verdade.E) Não é possível determinar sem um dicionário LUCIANÊS-PORTUGUÊS

2003. Carlinhos pensa num número ímpar positivo menor do que 100. Pe-drinho se dispõe a descobrir que número é esse fazendo a seguinte pergunta,quantas vezes forem necessárias: "O número que você pensou é maior, menorou igual a x ?". Note que x é um número que Pedrinho escolhe. Quantasperguntas desse tipo Pedrinho poderá ter que fazer até descobrir o númeropensado por Carlinhos?

2004. O produto dos números que aparecem nas alternativas incorretasdessa questão é um cubo perfeito. Assinale a alternativa correta.

Page 12: Alan Anderson 30 de janeiro de 2017 · 6.4.10 Áreas para encontrar razões entre ... façam o mesmo com os outros níveis e ... Agora esse texto visa ajudar estudantes que não sabem

12CAPÍTULO 1. PROBLEMAS DE LÓGICA E TEORIA DOS CONJUNTOS

A) 4 B) 8 C) 18 D) 54 E) 192

2004. O dono de uma loja empilhou vários blocos medindo 0, 8m×0, 8m×0, 8m no canto da loja e encostados numa parede de vidro que dá para a rua,conforme mostra a �gura abaixo. Quantos blocos no máximo, uma pessoade 1, 80m de altura que está do lado de fora da loja pode enxergar?

(FIGURA)Obs. Consideramos que uma pessoa pode enxergar uma caixa se conse-

gue ver uma pequena região de área positiva de sua superfície.

2004. Sobre uma mesa estão três caixas e três objetos, cada um em umacaixa diferente: uma moeda, um grampo e uma borracha. Sabe-se que

A caixa verde está à esquerda da caixa azul;A moeda está à esquerda da borracha;A caixa vermelha está à direita do grampo;A borracha está à direita da caixa vermelha.Em que caixa está a moeda?A) Na caixa vermelha .B) Na caixa verde.C) Na caixa azul.D) As informações fornecidas são insu�cientes para se dar uma resposta.E) As informações fornecidas são contraditórias.

2004. Um feirante vende batatas e, para pesar, utiliza uma balança dedois pratos, um peso de 1kg, um peso de 3kg e um peso de 10kg. Considere aseguinte a�rmação: "Este feirante consegue pesar (com uma pesagem) n qui-logramas de batatas". Quantos valores positivos de n tornam essa a�rmaçãoverdadeira, supondo que ele pode colocar pesos nos dois pratos?

A) 7 B) 10 C) 12 D)13 E)14

Page 13: Alan Anderson 30 de janeiro de 2017 · 6.4.10 Áreas para encontrar razões entre ... façam o mesmo com os outros níveis e ... Agora esse texto visa ajudar estudantes que não sabem

Capítulo 2

Combinatória e Probabilidade

Pré-requisito: princípio aditivo, princípio multiplicativo

2.1 Exercícios de Contagem

1998. Quantos números inteiros entre 10 e 1000 possuem seus dígitos emordem estritamente crescente? (Por exemplo, 47 e 126 são números destetipo; 52 e 566 não).

1999. Um gafanhoto pula exatamente 1 metro. Ele está em um ponto Ade uma reta, só pula sobre ela, e deseja atingir um ponto B dessa mesma retaque está a 5 metros de distância de A com exatamente 9 pulos. De quantasmaneiras ele pode fazer isso?

2000. Quantos números inteiros e positivos menores do que 1.000.000existem cujos cubos terminam em 1?

2000. Quantos números de três algarismos (que não começam com 0)possuem um algarismo que é a média aritmética dos outros dois?

2000. (FIGURA) Quantos são os retângulos que têm os pontos A e Bcomo vértices, e cujos vértices estão entre os pontos de interseção das 9 retashorizontais com as 9 retas verticais da �gura abaixo?

2001. Quantos números de dois algarismos não são primos nem múltiplos

13

Page 14: Alan Anderson 30 de janeiro de 2017 · 6.4.10 Áreas para encontrar razões entre ... façam o mesmo com os outros níveis e ... Agora esse texto visa ajudar estudantes que não sabem

14 CAPÍTULO 2. COMBINATÓRIA E PROBABILIDADE

de 2, 3 ou 5?

2001. Os números inteiros positivos de 1 a 1000 são escritos lado a lado,em ordem crescente, formando a seqüência 123456789101112131415 · · · 9991000.Nesta seqüência, quantas vezes aparece o grupo ”89” ?

2001. Um círculo é dividido, por 2n+ 1 raios, em 2n+ 1 setores congru-entes. Qual é o número máximo de regiões do círculo determinadas por estesraios e por uma reta?

2003. Cinco amigos, Arnaldo, Bernaldo, Cernaldo, Dernaldo e Ernaldo,devem formar uma �la com outras 30 pessoas. De quantas maneiras pode-mos formar esta �la de modo que Arnaldo �que na frente de seus 4 amigos.(Obs.: Os amigos não precisam �car em posições consecutivas.)

2004. Esmeralda, a digitadora, tentou digitar um número de seis alga-rismos, mas os dois algarismos 1 não apareceram (a tecla devia estar comdefeito). O que apareceu foi 2004. Quantos são os números de seis algarismosque ela pode ter tentado digitar?

2004. Com três algarismos distintos a, b e c, é possível formar 6 númerosde dois algarismos distintos. Quantos conjuntos {a, b, c} são tais que a somados 6 números formados é 484?

A) Um B) Dois C) Três D) Quatro E) Mais que quatro

2004. O arranjo a seguir, composto por 32 hexágonos, foi montado comvaretas, todas com comprimento igual ao lado do hexágono. Quantas vare-tas, no mínimo, são necessárias para montar o arranjo?

2004. Esmeralda escreveu (corretamente!) todos os números de 1 a 999,um atrás do outro:

12345678910111213?997998999.

Quantas vezes aparece o agrupamento ”21”, nesta ordem?

Page 15: Alan Anderson 30 de janeiro de 2017 · 6.4.10 Áreas para encontrar razões entre ... façam o mesmo com os outros níveis e ... Agora esse texto visa ajudar estudantes que não sabem

2.2. NÚMEROS BINOMIAIS 15

2.2 Números binomiais

2.2.1 Fatorial

Antes de tudo, um conceito bastante importante é o de fatorial. Dado umnúmero natural n escrevemos n! (n fatorial) para representar o produto dosinteiros positivos menores ou iguais a n, isto é, n! = n ·(n−1) ·(n−2)...3 ·2 ·1.Por razões práticas, de�nimos 0! = 1.

Esta de�nição é muito prática para representar valores em problemas decombinatória.

Exercício: Mostre que (n+ 1)! = (n+ 1) · n!.

Exercício: Calcule 1!, 2!, 3!, 4!, 5!, 6!, 7!, 8! (é útil lembrar pelo menos até6!).

Observe que

i) n · (n− 1) · ... · (k + 2) · (k + 1) =n!

k!.

ii) n · (n− 1) · ... · (n− k + 2) · (n− k + 1) =n!

(n− k)!.

Exemplo: De quantas maneiras podemos ordenar 7 crianças em uma�la?

Exemplo: Quantas quádruplas ordenadas podemos fazer com 4 elemen-tos distintos de {1, 2, ..., 9}?

Exemplo: Quantos subconjuntos com 5 elementos possui o conjunto{1, 2, ..., 9}.

2.2.2 Número Binomial

De�nição 1 Dados n ∈ N e k ∈ N ∪ {0}, de�nimos o número(nk

), lê-se

binomial de n e k, como sendo(n

k

)=

n!

(n− k)! · k!.

Page 16: Alan Anderson 30 de janeiro de 2017 · 6.4.10 Áreas para encontrar razões entre ... façam o mesmo com os outros níveis e ... Agora esse texto visa ajudar estudantes que não sabem

16 CAPÍTULO 2. COMBINATÓRIA E PROBABILIDADE

É costume chamar o número(nk

)de n escolhe k, pelo falto de ser a quan-

tidade de subconjuntos que fazemos escolhendo k elementos de um conjuntoque possui n elementos.

Proposição 1 (Propriedades dos números binomiais) Dados n ∈ N ek ∈ N ∪ {0} temos

i)(n+1k+1

)= n+1

k+1

(nk

);

ii)(n+1k

)= n+1

n−k+1

(nk

);

iii)(nk

)=(

nn−k

);

iv)(n+1k+1

)=(nk+1

)+(nk

).

Prova: A prova dessa proposição é trivial algebricamente pela de�niçãovia fatorial (Faça).

A seguir daremos provas combinatoriais dos itens iii) e iv), ilustrando ummétodo conhecido como contagem dupla, que consiste em contar uma coisade dois modos diferentes para se obter uma identidade.

iii) Suponha que queremos montar um conjunto A com k elementos de{1, 2, ..., n}, podemos fazer isso escolhendo k elementos para por em A, temos(nk

)maneiras de fazer isso, ou escolhendo n− k elementos para �carem fora

de A, temos(

nn−k

)maneiras de fazer isso. Assim

(nk

)=(

nn−k

).

iv) Suponha que queiramos fazer um subconjunto A com k+ 1 elementosdo conjunto {1, 2, ..., n+1}. Sabemos que podemos fazer isso de

(n+1k+1

)manei-

ras. Mas observe que para fazer isso, podemos por no conjunto A o elemento'n + 1' e em seguida escolher outros k elementos para por em A, temos

(nk

)maneiras de fazer isso, ou podemos não por o elemento 'n + 1', e montaro conjunto A apenas com elementos de {1, 2, ..., n},temos

(nk+1

)maneiras de

fazer isso. Assim, segue que(n+1k+1

)=(nk+1

)+(nk

). �

2.2.3 Justi�cando o nome

A seguir veremos porque esses números são chamados de números binomiais.Considere um binômio, ou seja, uma expressão do tipo x + y. O que

acontece se elevarmos essa expressão a n-ésima potência?

(x+ y)n = (x+ y)(x+ y)...(x+ y)

Page 17: Alan Anderson 30 de janeiro de 2017 · 6.4.10 Áreas para encontrar razões entre ... façam o mesmo com os outros níveis e ... Agora esse texto visa ajudar estudantes que não sabem

2.3. PRINCÍPIO DAS CASAS DOS POMBOS 17

Qual é o coe�ciente de xn nessa expressão? Para que obtenhamos o termoxkyn−k devemos multiplicar x em k dos parênteses, ou seja, temos n pa-rênteses para escolher em quais multiplicamos x (e os que não escolhermosmultiplicaremos y). Assim o coe�ciente de xkyn−k em (x+ y)n é

(nk

)donde

(x+ y)n =

(n

0

)xny0 +

(n

1

)xn−1y1 + ...+

(n

n− 1

)x1yn−1

(n

n

)x0yn.

2.3 Princípio das casas dos pombos

O princípio das casas dos pombos é um teorema muito famoso e muito simplesque diz que se temos mais pombos do que casas, e vamos por todos os pombosdentro das casas, alguma casa terá mais que um pombo. Quero deixar claroque o princípio também vale para gatos ou peixes. O princípio das casas dospombos é também conhecido como princípio das gavetas, é só trocar pombopor bolas e casa de pombo por gaveta.

Teorema 1 (Prinicípio das casas dos pombos) Se pomos k+ 1 pombosem k gaiolas, alguma gaiola terá pelo menos 2 pombos.

Em nome da liberdade dos animais não ponha pombos em gaiolas.

Prova:Suponha que tenhamos uma organização em que em cada gaiola pomos

menos que dois pombos, ou seja, uma quantidade ≤ 1, como temos k gaiolasteremos no total ≤ k pombos, mas isso contratria o fato de termos k + 1pombos.

O princípio pode ser escrito de maneiras mais gerais, um delas é

Teorema 2 (Prinicípio das casas dos pombos, segunda versão) Se po-mos nk+1 pombos em n gaiolas, alguma gaiola terá pelo menos k+1 pombos.

Prova: Exercício. Use um raciocínio similar ao da primeira versão.

O princípio das casas do pombos parece ridiculamente simples, e talvez oseja, mas isso não diminui o poder que ele possui. As aplicações são surpre-endentes. A seguir veremos alguns exemplos para que possamos ver porque

Page 18: Alan Anderson 30 de janeiro de 2017 · 6.4.10 Áreas para encontrar razões entre ... façam o mesmo com os outros níveis e ... Agora esse texto visa ajudar estudantes que não sabem

18 CAPÍTULO 2. COMBINATÓRIA E PROBABILIDADE

esses pombos merecem ser respeitados.

2000. Uma caixa contém 900 cartões, numerados de 100 a 999. Retiram-se ao acaso (sem reposição) cartões da caixa e anotamos a soma dos seusalgarismos. Qual é a menor quantidade de cartões que devem ser retiradosda caixa, para garantirmos que pelo menos três destas somas sejam iguais?

2.4 Jogos e Invariantes(Seção de exemplos)

1998. Hoje é sábado. Que dia da semana será daqui a 99 dias?

1998. Qual é o dígito das unidades do número 31998?

2000. Em um jogo de duas pessoas, os jogadores tiram, alternadamente,1, 2, 3, 4 ou 5 palitos de uma pilha que inicialmente tem 1000 palitos. Ganhao jogador que tirar o último palito da pilha. Quantos palitos o jogador quecomeça deve tirar na sua jogada inicial de modo a assegurar sua vitória?

2000. (FIGURA) Colocamos em ordem crescente os números escritos nascasas brancas do tabuleiro a seguir (estamos mostrando apenas as suas qua-tro primeiras linhas). Assim, por exemplo, o nono número da nossa lista é14. Qual é o 2000o número da nossa lista?

2003. Camila e Lara estão disputando o seguinte jogo num tabuleiro4× 4: Camila marca algumas casas do tabuleiro e informa à Lara o númerode casas marcadas na vizinhança de cada casa do tabuleiro. Neste jogo, duascasas distintas são consideradas vizinhas se possuem um lado ou um canto(vértice) em comum. Lara deve descobrir quais casas foram marcadas porCamila. Após marcar algumas casas, Camila passou para Lara o seguintetabuleiro:

(FIGURA)Qual é número de casas marcadas?

2003.Em um quadro negro escreve-se o número 1. As únicas alteraçõespermitidas são substituí-lo pelo seu dobro ou pelo seu quadrado. Qual é omaior número que pode ser obtido após efetuarmos 2003 alterações?

Page 19: Alan Anderson 30 de janeiro de 2017 · 6.4.10 Áreas para encontrar razões entre ... façam o mesmo com os outros níveis e ... Agora esse texto visa ajudar estudantes que não sabem

2.5. PROBABILIDADE 19

2.5 Probabilidade

Falando sem rigor, no nosso contexto a probabilidade de que algo aconteçaserá a razão entre o "tamanho do ambiente favorável" pelo "tamanho doambiente das possíbilidades".

Exemplo 1

Se queremos saber a probabilidade de escolher um número ímpar no con-junto {1, 2, 3, 4, 5}, o ambiente favorável será {1, 3, 5}, que tem três elemen-tos, ou tamanho três no nosso sentido informal. Assim a probabilidade de-sejada será 3

5.

Exemplo 2

Qual é a probabilidade de escolher um número no intervalo [0, 25] e eleser menor 18, 7 e maior que 13?

Nesse caso o conjunto total tem comprimento 25 enquanto que o conjuntoonde estão os números que queremos tem comprimento 5, 7, logo a probabili-dade será 5,7

25, ou se quisermos escrever de modo um pouquinho mais elegante

57250

.

Exemplo 3

Considere um qudrado ABCD de lado 2 e um semicírculo de raio 1 comdiâmetro AB dentro do quadrado. Qual é a probabilidadde de escolhermosum ponto do quadrado e ele estar dentro dos semicírculo?

Neste caso, temos que o "tamanho total"a ser considerado é a área doquadrado, enquanto que o favorável será a área do semi-círculo. Assim aprobabilidade será

π2

4= π

8.

Agora que já demos a idéia geral do que será a probabilidade iremospartir para exemplos que podem ser um pouco mais confusos a primeiravista. Olhando para cada "tipo"de ambiente: conjuntos �nitos, segmentos,superfícies planas, etc.

Page 20: Alan Anderson 30 de janeiro de 2017 · 6.4.10 Áreas para encontrar razões entre ... façam o mesmo com os outros níveis e ... Agora esse texto visa ajudar estudantes que não sabem

20 CAPÍTULO 2. COMBINATÓRIA E PROBABILIDADE

2.5.1 Probabilidade Discreta

Consideraremos como probabilidade discreta a probabilidade em conjuntos�nitos. A probabilidade aqui será simplesmente

P (algo acontecer) =número de casos favoráveisnúmero de casos possíveis

.

Em essência não é necessário estudar combinatória pra estudar proba-bilidade. Essas duas coisas são apresentadas juntas pelo fato de que sema ferramenta da combinatória �caria difícil contar as quantidades de casosfavoráveis e de casos possíveis. Como a de�nição é incrivelmente simples,procederemos por meio de exemplos.

Exemplo Um dado comum é lançado. Qual é a probabilidade da facevoltada para cima ser um número primo?

O conjunto das possibilidade é {1, 2, 3, 4, 5, 6}, que tem 6 elementos. Oconjunto dos casos favoráveis é {2, 5}, que possui dois elementos. Logo aprobabilidade desejada é 2

6= 1

3.

Exemplo Duas moedas comuns são lançadas simultaneamente. Qual é aprobabilidade das duas moedas darem o mesmo resultado.

Os possíveis resultados para as faces a mostra das moedas são cara quesimbolizaremos por C, ou coroa, que simbolizaremos por K. Assim, o con-junto das possibilidades nesse caso são (C,C), (C,K), (K,C) ou (K,K). Oscasos favoráveis aqui são então (C,C) ou (K,K), assim temos duas possibi-lidades em quatro e a probabilidade desejtada é 1

2.

Um tipo de coisa que pode deixar uma pessoa confusa com respeito aprobabilidade é o seguinte tipo de problema:

Exemplo Sejam A = {1, 2, 3, 4, 5} e B = {2, 4, 6, 8}. Qual é a probabili-dade de se escolher um elemento de A e ele pertencer a B?

A confusão surge quando se considera B de casos favoráveis. Nesse caso,deve analizar quantos elementos de A estão em B e não simplesmente quan-tos elementos estão em B. Os casos favoráveis, são sempre um subconjunto

Page 21: Alan Anderson 30 de janeiro de 2017 · 6.4.10 Áreas para encontrar razões entre ... façam o mesmo com os outros níveis e ... Agora esse texto visa ajudar estudantes que não sabem

2.5. PROBABILIDADE 21

dos casos possíveis. E nesse caso a probabilidade é 25e não 3

5como poderia

imaginar um leitor desatento.

Exemplo Em uma urna há duas moedas aparentemente iguais. Uma de-las é uma moeda comum, com uma cara e uma coroa. A outra, no entanto,é uma moeda falsa, com duas caras. Suponhamos que uma dessas moedasseja sorteada e lançada. Se o resultado do lançamento é cara, qual é a pro-babilidade de que a moeda sorteada tenha sido a comum?

Aqui é natural que se cometa a confusão de que a probabilidade de esco-lher a moeda comum é 1

2e por isso a resposta é 1

2. De fato a probabilidade

de escolher a moeda comum é 12. No entanto temos uma informação extra de

que depois do lançamento obtvemos uma cara. Assim nossas possibilidadessão:

1. Essa é a cara da moeda comum2. Essa é uma cara da moeda com duas caras3. Essa é a outra cara da moeda com duas carasAssim, o conjunto de casos possíveis tem 3 elementos, enquanto que o de

casos favoráveis tem apenas 1. Portanto a resposta é na verdade 13.

2.5.2 Probabilidade Geométrica

Quando queremos calcular a probabilidade de escolher um ponto numa partede um segmento, basta dividir o comprimento da região favorável pelo daregião possível. Analogamente para superfície, divide a área da região favo-rável pela área da região possível. Isso é uma de�nição. Como no caso deprobabilidade discreta iremos o tema será apresentado por meio de exemplos,já que só temos essa de�nição.

Exemplo Seja A um ponto num plano. Suponha que escolhemos umponto B nesse plano a uma distância a A é ≤ 2. Qual é a probabilidade deque B esteja a uma distância ≤ 1 de A?

O conjunto dos pontos a uma distância ≤ d de A é o círculo de centro A eraio d, cuja área como sabemos é πd2. Assim a área possível é 4π, enquantoque a área favorável é π, logo a probabilidade é 1

4.

Page 22: Alan Anderson 30 de janeiro de 2017 · 6.4.10 Áreas para encontrar razões entre ... façam o mesmo com os outros níveis e ... Agora esse texto visa ajudar estudantes que não sabem

22 CAPÍTULO 2. COMBINATÓRIA E PROBABILIDADE

O tipo de confusão com casos favoráveis que ocorre com os problemas deprobabilidade em conjuntos �nitos pode ocorrer também em probabilidadegeométrica. Vejamos a seguir um exemplo semelhante ao que vimos na pri-meira subseção.

ExemploConsidere um qudrado ABCD de lado 2 e um círculo Γ de raio 1 com di-

âmetro AB. Qual é a probabilidadde de escolhermos um ponto do quadradoe ele estar dentro do círculo?

Neste caso, temos que o "tamanho total"a ser considerado é a área doquadrado, enquanto que o favorável será a área da parte de Γ que está den-tro de ABCD. Assim a probabilidade será

π2

4= π

8, e não π

4como se poderia

imaginar..

2.5.3 Exercícios da OBM

2001. Uma rifa foi organizada entre os 30 alunos da turma do Pedro. Paratal, 30 bolinhas numeradas de 1 a 30 foram colocadas em uma urna. Umadelas foi, então, retirada da urna. No entanto, a bola caiu no chão e se per-deu e uma segunda bola teve que ser sorteada entre as 29 restantes. Qual aprobabilidade de que o número de Pedro tenha sido o sorteado desta segundavez?

2002. Duas pessoas vão disputar uma partida de par ou ímpar. Elas nãogostam do zero e, assim, cada uma coloca 1, 2, 3, 4 ou 5 dedos com igual pro-babilidade. Qual é a probabilidade de que a pessoa que escolheu par ganhe?

2003. Beatriz, Isabele e Nicole estão disputando um jogo fazendo lança-mentos sucessivos com uma moeda. Beatriz ganha se, em dois lançamentosconsecutivos, o primeiro resultar cara e o segundo coroa. Isabele ganha seforem obtidas duas coroas em dois lançamentos consecutivos, e Nicole ganhase forem obtidas duas caras em dois lançamentos consecutivos. Elas fazem oslançamentos até que uma das jogadoras seja vencedora. Qual(is) jogadora(s)possuem menos chances de ganhar o jogo?

Page 23: Alan Anderson 30 de janeiro de 2017 · 6.4.10 Áreas para encontrar razões entre ... façam o mesmo com os outros níveis e ... Agora esse texto visa ajudar estudantes que não sabem

2.5. PROBABILIDADE 23

2004. Dois cubos têm faces pintadas de ocre ou magenta. O primeirocubo tem cinco faces ocres e uma face magenta. Quando os dois cubos sãolançados, a probabilidade de as faces viradas para cima dos dois cubos seremda mesma cor (sim, ocre e magenta são cores!) é 1/2. Quantas faces ocrestem o segundo cubo?

Page 24: Alan Anderson 30 de janeiro de 2017 · 6.4.10 Áreas para encontrar razões entre ... façam o mesmo com os outros níveis e ... Agora esse texto visa ajudar estudantes que não sabem

24 CAPÍTULO 2. COMBINATÓRIA E PROBABILIDADE

Page 25: Alan Anderson 30 de janeiro de 2017 · 6.4.10 Áreas para encontrar razões entre ... façam o mesmo com os outros níveis e ... Agora esse texto visa ajudar estudantes que não sabem

Capítulo 3

Álgebra

Pré-requisito: Sistemas de equações, básico de polinômios, básico sobre ine-quações/desigualdades, básico de funções.

3.1 Produtos Notáveis e Fatorações Notáveis

3.1.1 Lista de produtos notáveis

A seguir listamos os produtos notáveis básicos.Quadrado da Soma:(a+ b)2 = a2 + 2ab+ b2

(a+ b+ c)2 = a2 + b2 + c2 + 2ab+ 2bc+ 2caQuadrado da diferença:(a− b)2 = a2 − 2ab+ b2

Cubo da soma(a+ b)3 = a3 + 3a2b+ 3ab2 + b3

Cubo da diferença(a− b)3 = a3 − 3a2b+ 3ab2 − b3Diferença de quadradosa2 − b2 = (a+ b)(a− b)Soma dos cubosa3 + b3 = (a+ b)(a2 − ab+ b2)Diferença dos cubosa3 − b3 = (a− b)(a2 + ab+ b2)

25

Page 26: Alan Anderson 30 de janeiro de 2017 · 6.4.10 Áreas para encontrar razões entre ... façam o mesmo com os outros níveis e ... Agora esse texto visa ajudar estudantes que não sabem

26 CAPÍTULO 3. ÁLGEBRA

Identidade de Sophi Germaina4 + 4b4 = (a2 + 2b2 + 2ab)(a2 + 2b2 − 2ab)

Exercício Veri�que cada identidade acima.

2002. Se xy = 2 e x2 + y2 = 5, então quanto vale x2

y2+ y2

x2?

2004. Sejam

a =12

2+

22

3+

32

5+ · · ·+ 10012

2001

b =12

3+

22

5+

32

7+ · · ·+ 10012

2003.

Qual é o inteiro mais próximo de a− b?

3.1.2 Completando Quadrados

Quando se deseja resolver uma equação de segundo grau, ou simpli�car umexpressão do tipo ax2 + bx + c em geral é muito útil fazer o que chamamosde completar quadrado. O propósito de completar quadrado é obter umaexpressão do tipo (αx+β)2 + γ, se a ≥ 0 ou do tipo −(αx+β)2 + γ se a < 0(com α, β, γ não necessariamente positivos). Esse tipo de coisa facilita naresolução de diversos problemas, entre eles a equação do segundo grau.

Começaremos com exemplos ilustrativos e depois iremos para o caso geral.

Exemplo Simpli�car a expressão x2 + 8x− 1.

Queremos escrever x2+8x−1 = (αx+β)2+γ. Observe que (αx+β)2+γ =αx2 + 2αβx+ β2 + γ. Assim queremos que

αx2 + 2αβx+ β2 + γ = x2 + 8x− 1.

A resposta a ser obtida não precisa ser única, apenas queremos umaresposta. Assim poderíamos considerar α = 1 para que os termos que acom-panham x2 se igualem. Observe que se tomamos α = −1 o mesmo ocorre,mas tomaremos o valor 1 por simplicidade.

Se α = 1 temos

Page 27: Alan Anderson 30 de janeiro de 2017 · 6.4.10 Áreas para encontrar razões entre ... façam o mesmo com os outros níveis e ... Agora esse texto visa ajudar estudantes que não sabem

3.2. BINÔMIO DE NEWTON 27

x2 + 2βx+ β2 + γ = x2 + 8x− 1⇒ 2βx+ β2 + γ = 8x− 1.

Agora vamos determinar β. De modo análogo queremos que 2β = 8, paraigualarmos os coe�cientes do termo x. Logo temos β = 4 e obtemos

2 · 4x+ 42 + γ = 8x− 1⇒ 16 + γ = −1

Finalmente escolhemos o γ adequado, que será então γ = −17.Assim podemos escrever a expressão x2 + 8x− 1 = (x+ 4)2 − 17.

Exercício Simpli�que, de modo análogo ao caso acima, a expressão−x2 + 12x− 13.

3.2 Binômio de Newton

Vimos acima como calcular (x+ y)n para n = 2 e n = 3. A seguir iremos vercomo fazer isso pra n qualquer.

Teorema 3

(x+ y)n =

(n

0

)xny0 +

(n

1

)xn−1y1 + ...+

(n

n− 1

)x1yn−1 +

(n

n

)x0yn.

Prova: Indução. Use que(n+1p+1

)=(np+1

)+(np

).

3.3 Equação do Segundo Grau

O que faremos a seguir ilustra uma técnica conhecida como completar qua-drados. A fórmula da solução da equação do segundo gra já é uma velhaconhecida, o que essa seção visa é mostrar como usar a ideia de completarquadrado e deduzir a fórmula, conhecida no Brasil como "fórmula de Bhás-kara".

Sabemos facilmente resolver alguns tipos de equações do segundo grau.

Page 28: Alan Anderson 30 de janeiro de 2017 · 6.4.10 Áreas para encontrar razões entre ... façam o mesmo com os outros níveis e ... Agora esse texto visa ajudar estudantes que não sabem

28 CAPÍTULO 3. ÁLGEBRA

Exemplo 1.

As soluções da equação x2 = γ com γ ≥ 0 são x1 =√γ e x2 = −√γ.

A equação x2 = γ com γ < 0 não temos solução real.

Exemplo 2.

Resolva a (αx+ β)2 = γ, com α 6= 0, γ > 0.

Solução: (αx + β)2 = γ ⇐⇒ (αx + β) = ±√γ ⇐⇒ αx = −β ± √γ.Ou seja, as soluções são x1 =

−β+√γα

ou x2 =−β−√γ

α.

Agora vamos analizar o caso geral, no qual nos é dado uma equação dosegundo grau.

Exemplo 3.

Sejam a, b, c números reais, com a 6= 0. Encontre as soluções da equação

ax2 + bx+ c = 0.

Solução: Nosso objetivo será reduzir essa equção a uma do tipo do Exem-plo 2. Observe que (αx + β)2 = α2x2 + 2αβx + β2. Como na equação quedesejamos resolver não sabemos se a > 0, para escrever a = α2, o que fa-remos é multiplicar a equação por a, obtendo a2x2 + abx + ac = 0.Fazemosa2x2 = α2x2, faremos então α = a.

O segundo passo será investigar o valor de 2αβx = 2aβx. Observe quecomo esse termo é o que acompanha x devemos ter 2aβx = abx, portanto2β = b, ou seja, β = b

2.

O passo �nal é olhara para β2. Veja que nesse caso devemos ter β2 = b2

4,

e para termos uma expressão idêntica a um quadrado da soma, iremos somarb2

4e obviamente temos que subtrair o mesmo valor, pois não podemos alterar

o valor da expressão. Assim obtemos que a nossa equação é equivalente a

Page 29: Alan Anderson 30 de janeiro de 2017 · 6.4.10 Áreas para encontrar razões entre ... façam o mesmo com os outros níveis e ... Agora esse texto visa ajudar estudantes que não sabem

3.3. EQUAÇÃO DO SEGUNDO GRAU 29

a2x2 + abx+ b2

4− b2

4+ ac = 0, agora vamos fazer conta:

ax2 + bx+ c = 0

⇐⇒ a2x2 + abx+ ac = 0

⇐⇒ a2x2 + abx+b2

4− b2

4+ ac = 0

⇐⇒ a2x2 + abx+b2

4=b2

4− ac

⇐⇒(ax+

b

2

)2

=b2 − 4ac

4

Assim, se b2 − 4ac < 0, a equação não tem solução, pois o lado direito seránegativo, e um número real elevado quadrado não pode ser negativo. Assim,se b2 − 4ac ≥ 0 temos

ax2 + bx+ c = 0

⇐⇒(ax+

b

2

)2

=b2 − 4ac

4

⇐⇒(ax+

b

2

)=±√b2 − 4ac

2

⇐⇒ ax = − b2±√b2 − 4ac

2

⇐⇒ ax =−b±

√b2 − 4ac

2

⇐⇒ x =−b±

√b2 − 4ac

2a.�

Exercício Encontre em função dos parâmetros, os valores da soma e doproduto das raízes de uma equação do segundo grau.

1999. Sendo a 6= b e b 6= 0, sabe-se que as raízes da equação x2+ax+b = 0são exatamente a e b. Então,quanto vale a− b?

1998. Qual é a diferença entre a maior raiz e a menor raiz da equação(2x− 45)2 − (x− 21)2 = 0?

2001. Qual é a soma dos valores reais de x tais que x2 + x+ 1 = 156x2+x

?

Page 30: Alan Anderson 30 de janeiro de 2017 · 6.4.10 Áreas para encontrar razões entre ... façam o mesmo com os outros níveis e ... Agora esse texto visa ajudar estudantes que não sabem

30 CAPÍTULO 3. ÁLGEBRA

2002. Seja α a maior raiz de x2 + x− 1 = 0. Qual é o valor de α5 − 5α?

2003. O grá�co de y = x2 − 5x + 9 é rodado 180o em torno da origem.Qual é a equação da nova curva obtida?

2004. Qual é conjunto das raízes reais da equação√x+ 2

√x− 1 +√

x− 2√x− 1 = 2?

3.4 Polinômios

Suporemos que o leitor sabe o que é um polinômio. Mas �ngiremos queestamos explicando algo abaixo e depois enunciaremos resultados de fato.Rasamente falando um polinômio é uma função p : R → R que leva x emp(x) = anx

n + ...+ a1x+ a0.

De�nição 2 Uma raiz de um polinômio p(x) é um número a tal que p(a) =0.

Observe que resolver uma equação do primeiro ou do segundo grau é omesmo que encontrar as raízes de um polinômio de primeiro ou de segundograu.

Teorema 4 (D'Alembert) Se x é raíz de p(x), então existe um polinômioq(x) tal que p(x) = (x− a)q(x).

1998. Qual é a soma das raízes reais de x3 + 3x2 + 3x− 1 = 0?

Teorema 5 Se p(x) é um polinômio com grau ímpar e coe�cientes reais,então p(x) possui uma raíz real.

Teorema 6 (Fundamental da Álgebra) Todo polinômio com coe�cien-tes complexos (em particular com coe�cientes reais) possui pelo menos umaraíz complexa.

A demonstração desse teorema, cujo enunciado é bastante simples re-quer conhecimentos superiores de matemática (onde de�nimos conhecimentosuperior como aquele que se obtém no ensino superior).

Page 31: Alan Anderson 30 de janeiro de 2017 · 6.4.10 Áreas para encontrar razões entre ... façam o mesmo com os outros níveis e ... Agora esse texto visa ajudar estudantes que não sabem

3.5. DESIGUALDES 31

Teorema 7 Se p(x) é um polinômio de grau n, então podemos escreverp(x) = an(x − x1)(x − x2) · · · (x − xn), onde an é o coe�ciente líder de pe x1, x2, · · · , xn são as raízes (não necessariamente distintas de p(x)).

Prova: Use o Teorema Fundamental da Álgebra e indução.

Teorema 8 Sejam p(x) = anxn + an−1x

n−1 + · · ·+ a1x+ a0. Então a soma

das raízes de p é igual a −an−1an

e o produto das raízes de p é igual aa0an

.

Prova: Use o teorema acima.

OBM

2000. Seja P (x) = a2000x2000 + a19999x

1999 + a1998x1998 + · · · + a1x + a0.

Então a2000 + a1998 + a1996 + · · ·+ a0 é igual a

A)P (1)− P (−1)

2B)P (1) + P (−1)

2C) P (2000) +P (1998) + · · ·+P (0)

D)P (0) · P (1) E) P (−1) · P (1)

3.5 Desigualdes

3.5.1 Exercícios de aquecimento

OBMF1. Quantos são os números inteiros x que satisfazem à inequação3 <√x < 7?

2000. Sejam a e b números reais positivos tais quea

b< 1. Então

a+ 1

b+ 1

A) é igual aa

b+ 1. B) é igual a

a

b. C) é menor que

a

b.

D) é maior quea

bmas menor que 1. E) pode ser maior que 1.

2002. Uma escola vai organizar um passeio ao zoológico. Há duas opçõesde transporte. A primeira opção é alugar "vans": cada van pode levar até6 crianças e seu aluguel custa R$60, 00. A segunda opção é contratar umaempresa para fazer o serviço: a empresa usa ônibus com capacidade para 48crianças e cobra R$237, 00, mais R$120, 00 por ônibus utilizado. A escoladeve preferir a empresa de ônibus se forem ao passeio pelo menos N crianças.

Page 32: Alan Anderson 30 de janeiro de 2017 · 6.4.10 Áreas para encontrar razões entre ... façam o mesmo com os outros níveis e ... Agora esse texto visa ajudar estudantes que não sabem

32 CAPÍTULO 3. ÁLGEBRA

Qual é o valor de N?

2002. Vamos provar que 4 é maior que 4. Sejam a e b dois números taisque a > 4 e a = b.

1) Vamos subtrair 4 dos dois termos desta equação:a = b

a− 4 = b− 4

2) Colocamos −1 em evidência no segundo membro da equação:a− 4 = −1(−b+ 4)

a− 4 = −1(4− b)3) Elevamos ambos os termos da equação ao quadrado:(a− 4)2 = [−1 · (4− b)]2(a− 4)2 = (−1)2(4− b)2(a− 4)2 = 1 · (4− b)2(a− 4)2 = (4− b)24) Extraímos a raiz quadrada dos dois membros da equação:(a− 4)2 = (4− b)2a− 4 = 4− b5) Como a = b, substituímos b por aa− 4 = 4− a6) Resolvemos a equação:a− 4 = 4− a2a = 8

a = 4

Como escolhemos a tal que a > 4, chegamos à inacreditável conclusão deque 4 > 4. Qual é o erro?

3.5.2 MA ≥ MG

Como sabemos se x ∈ R, temos que x2 ≥ 0. Essa desigualdade será a baseda prova da desigualdade entre as médias aritimética e geométrica.

Exercício Use x = (√a−√b), com a, b > 0 para provar que

a+ b

2≥√ab.

Page 33: Alan Anderson 30 de janeiro de 2017 · 6.4.10 Áreas para encontrar razões entre ... façam o mesmo com os outros níveis e ... Agora esse texto visa ajudar estudantes que não sabem

3.6. FUNÇÃO PARTE INTEIRA 33

Exercício Aplique a desigualdade acima duas vezes para mostrar que sea, b, c, d > 0 então

a+ b+ c+ d

4≥ 4√abcd.

Exercícioa) Use d = 3

√abc

b) Use d = a+b+c3

na desigualdade acima e mostre que se a, b, c > 0, então

a+ b+ c

3≥ 3√abc.

Exercício Mostre que sin(x) · cos(x) ≤ 12.

1998. Quais são os valores reais de x que satisfazem a inequação

√x+

√1

x≤ 2?

2000. Se x e y são números reais positivos, qual dos números a seguir é omaior?

A) xy B) x2 + y2 C) (x+ y)2 D)x2 + y(x+ y) E)x3 + y3

x+ y

Teorema 9 (Desigualdade MA ≥ MG) Sejam a1, ..., an > 0, então

a1 + a2 + ...+ ann

≥ n√a1a2...an.

Prova: Exercício. Indução forte: suponha que provou pra todos os valo-res ≤ 2k, prove para n = 2k + 2 e em seguida para n = 2k + 1 (use as idéiasde n = 4 e depois de n = 3).

3.6 Função Parte Inteira

De�nição 3 A parte inteira de um número x ∈ R, bxc, é o maior inteiron tal que n ≤ x < n+ 1.

Page 34: Alan Anderson 30 de janeiro de 2017 · 6.4.10 Áreas para encontrar razões entre ... façam o mesmo com os outros níveis e ... Agora esse texto visa ajudar estudantes que não sabem

34 CAPÍTULO 3. ÁLGEBRA

Podemos então ver parte inteira como uma função b·c : R → Z que levaum número x na parte inteira de x, bxc, ou seja, no maior inteiro n tal quen ≤ x < n+ 1.

Exemplos:1. Observe que se x > 0, bxc é o número antes da vírgula na representação

decimal de x, por exemplo b2, 5c = 2, bπc = 3.2. No entanto se x < 0 a parte inteira não é o número que vem antes

da vírgula, e sim esse número subtraído de 1, pois bxc ≤ x,∀x ∈ R. Assimtemos por exemplo que b−2, 5c = −3.

Em geral, quando se deseja fazer cálculos envolvendo parte inteira, trabalha-se com a desigualdade da de�nição, ou seja, com o fato de que bxc ≤ x <bxc+ 1 e com o fato de que bxc ∈ Z.

A seguir veremos algumas propriedades e �cará explícito o que queremosdizer com o parágrafo anterior.

Proposição 2 (Propriedades da parte inteira) Dado x, y ∈ R e n ∈ Nvalem as seguintes expressões

i) bxc+ byc ≤ bx+ yc;ii) bn · xc ≥ n · bxc.

Teorema 10 (Fórmula de Polignac) Seja p um número primo. A maiorpotência m de p tal que pm|n! é dada por

m =∞∑j=1

⌊n

pj

⌋.

2000 . Qual é número de inteiros positivos x para os quais bx 12 c+ bx 1

3 c =10?

2003. Calcule⌊

22003 + 32003

22001 + 32001

⌋.

3.7 Recorrências

3.7.1 Exercícios da OBM

2003. Considere a seqüência oscilante: 1, 2, 3, 4, 5, 4, 3, 2, 1, 2, 3, 4, 5, 4, 3, 2, 1, 2, 3, 4, · · · .Qual é o 2003o termo desta seqüência?

Page 35: Alan Anderson 30 de janeiro de 2017 · 6.4.10 Áreas para encontrar razões entre ... façam o mesmo com os outros níveis e ... Agora esse texto visa ajudar estudantes que não sabem

3.7. RECORRÊNCIAS 35

3.7.2 PA e PG

Os primeiros exemplos de recorrências que nos são apresentados na escola (etalvez os únicos) são as progressões aritméticas e geométricas (PA e PG).

Começaremos resolvendo o clássico problema de calcular a soma dos nprimeiro números naturais. Esse problema é lendário, procure sua história enão conseguirá dormir em paz hoje.

Exemplo: Calcule S = 1 + 2 + 3 + ...+ 100.

Solução de Gauss: Observe que S = 100 + 99 + ... + 3 + 2 + 1. Entãosomando "verticalmente"temos

S = 1 + 2 + · · ·+ 99 + 100

S = 100 + 99 + · · ·+ 2 + 1

2S = 101 + 101 + · · ·+ 101 + 101 = 100 · 101 = 10100

Logo S = 5050.

Exercício Calcule Sn = 1 + 2 + · · ·+ n.

De�nição 4 Uma PA é uma sequência com um termo inicial a1 e tal quepara cada n ∈ N, an+1 = an + r, onde r ∈ R.

Exercício a) Considere a PA com a1 = 7 e r = 4. Calcule S23 =a1 + · · ·+ a23.

b) No caso acima, determine Sn = a1 + · · ·+ an.c)Calcule a soma dos n primeiros termos de uma PA de razão r e termo

inicial a1: Sn = a1 + ...+ an.

De�nição 5 Uma PG é uma sequência com um termo inicial a0 e tal quepara cada n ∈ N, an = an−1q, onde q ∈ R.

Exemplo: Seja a ∈ R. Calcule Sn = 1 + a+ a2 + a3 + ...+ an.

Inicialmente, se a = 1 o problema é trivial. Suponha que a 6= 1. Observeque

Sn + an+1 = 1 + a+ a2 + a3 + ...+ an + an+1 = 1 + aSn,

logo

Page 36: Alan Anderson 30 de janeiro de 2017 · 6.4.10 Áreas para encontrar razões entre ... façam o mesmo com os outros níveis e ... Agora esse texto visa ajudar estudantes que não sabem

36 CAPÍTULO 3. ÁLGEBRA

Sn − aSn = 1− an+1 ⇒ Sn =1− an+1

1− a.

Exercício: Calcule a soma dos termos de uma PG: Sn = a0+a1+ ...+an.

Exercício: Uma SAG, ou sequência aritmético-geométrica é uma sequên-cia com um termo inicial a0 e tal que an = an−1 ·q+r, onde q, r ∈ R. Encontrea soma dos n primeiros termos da SAG.

3.7.3 Equação Característica (escrever)

3.7.4 Pequenas modi�cações resolvem problemas (seçãode exemplos)

(Cone Sul 1996)Considere uma sequência de números reais de�nida por:

an+1 = an +1

anpara n = 0, 1, 2, ...

Demostre que, qualquer que seja o número real positivo a0, tem-se que a1996é maior que 63.

(Moldávia 1998)

A sequência (an) , n ∈ N veri�ca as relações a1 =1

2, an =

an−12nan−1 + 1

para todo número natural n > 1 . Calcule a1 + a2 + ...+ a1998.

(Lista Cone Sul 2016)Seja {an}n≥1 uma sequência tal que a1 = 4 e, para todo n inteiro positivo:

an+1 =an2

+2

an.

Determine an em função de n.

Page 37: Alan Anderson 30 de janeiro de 2017 · 6.4.10 Áreas para encontrar razões entre ... façam o mesmo com os outros níveis e ... Agora esse texto visa ajudar estudantes que não sabem

3.8. EQUAÇÕES E SISTEMAS(SEÇÃO DE EXEMPLOS) 37

3.8 Equações e sistemas(Seção de exemplos)

1998. A média aritmética de seis números é 4. Quando acrescentamos umsétimo número, a nova média é 5. Qual número que foi acrescentado?

1998. Barcas vão do Rio a Niterói em 25 minutos e lanchas fazem a via-gem em 15 minutos. A que horas a barca que partiu do Rio às 10h01min éalcançada pela lancha que saiu do Rio às 10h07min?

1999. Um pequeno caminhão pode carregar 50 sacos de areia ou 400 tijo-los. Se foram colocados no caminhão 32 sacos de areia, quantos tijolos podeainda ele carregar?

1999. Uma bola de futebol é feita com 32 peças de couro. 12 delas sãopentágonos regulares e as outras 20 são hexágonos também regulares. Oslados dos pentágonos são iguais aos dos hexágonos de forma que possam sercosturados. Cada costura une dois lados de duas dessas peças. Quantas sãoas costuras feitas na fabricação de uma bola de futebol?

1999. Pedro saiu de casa e fez compras em quatro lojas, cada uma numbairro diferente. Em cada uma gastou a metade do que possuía e a seguir,ainda pagou R$ 2, 00 de estacionamento. Se no �nal ainda tinha R$8, 00, quequantia tinha Pedro ao sair de casa?

1999. Em um aquário há peixes amarelos e vermelhos: 90% são amarelose 10% são vermelhos. Uma misteriosa doença matou muitos peixes amarelos,mas nenhum vermelho. Depois que a doença foi controlada veri�cou-se queno aquário, 75% dos peixes vivos eram amarelos. Aproximadamente, queporcentagem dos peixes amarelos morreram?

1999. Hoje, 12/6/1999, Pedro e Maria fazem aniversário. No mesmodia em 1996, a idade de Pedro era 3/4 da idade de Maria. No mesmo diaem 2002, a idade de Pedro será igual à de Maria quando ele tinha 20 anos.Quantos anos Maria está fazendo hoje?

1999. Um retângulo ABCD está dividido em quatro retângulos menores.As áreas de três deles estão na �gura abaixo. Qual é a área do retânguloABCD?

Page 38: Alan Anderson 30 de janeiro de 2017 · 6.4.10 Áreas para encontrar razões entre ... façam o mesmo com os outros níveis e ... Agora esse texto visa ajudar estudantes que não sabem

38 CAPÍTULO 3. ÁLGEBRA

2000. Alberto, Beatriz e Carlos correm numa pista circular. Todos saemao mesmo tempo e do mesmo lugar, cada um desenvolvendo velocidade cons-tante. Alberto e Beatriz correm no mesmo sentido. Correndo no sentidooposto, Carlos encontra Alberto, pela primeira vez, exatamente 90 segundosapós o início da corrida e encontra Beatriz exatamente 15 segundos depois.Quantos segundos são necessários para que Alberto ultrapasse Beatriz pelaprimeira vez?

2000. Quantos são os números inteiros de 2 algarismos que são iguais aodobro do produto de seus algarismos?

2000. Dois nadadores, inicialmente em lados opostos de uma piscina, co-meçam simultaneamente a nadar um em direção ao outro. Um deles vai deum lado a outro da piscina em 45 segundos e o outro em 30 segundos. Elesnadam de um lado para outro por 12 minutos, sem perder qualquer temponas viradas. Quantas vezes eles passam um pelo outro (indo no mesmo sen-tido ou em sentidos opostos) durante este tempo, contando as vezes em quese encontram nos extremos da piscina?

2000. De Itacimirim a Salvador, pela estrada do Coco, são 60 km. Às11 horas,a 15 km de Salvador, dá-se um acidente que provoca um engarrafa-mento, que cresce à velocidade de 4 km/h, no sentido de Itacimirim. A quehoras, aproximadamente, devemos sair de Itacimirim para chegar a Salvadorao meiodia, sabendo que viajamos a 60 km/h, exceto na zona de engarrafa-mento, onde a velocidade é 6 km/h?

2000. O emir Abdel Azir �cou famoso por vários motivos. Ele teve maisde 39 �lhos, incluindo muitos gêmeos. De fato, o historiador Ahmed Aaba�rma num dos seus escritos que todos os �lhos do emir eram gêmeos du-plos, exceto 39; todos eram gêmeos triplos, exceto 39; todos eram gêmeosquádruplos, exceto 39. Qual é o número de �lhos do emir?

2001. Uma pêra tem cerca de 90% de água e 10% de matéria sólida. Umprodutor coloca 100 quilogramas de pêra para desidratar até o ponto em quea água represente 60% da massa total. Quantos litros de água serão evapo-rados? (lembre-se: 1 litro de água tem massa de 1 quilograma).

Page 39: Alan Anderson 30 de janeiro de 2017 · 6.4.10 Áreas para encontrar razões entre ... façam o mesmo com os outros níveis e ... Agora esse texto visa ajudar estudantes que não sabem

3.8. EQUAÇÕES E SISTEMAS(SEÇÃO DE EXEMPLOS) 39

2001. (FIGURA) Um serralheiro tem 10 pedaços de 3 elos de ferro cadaum, mostrados abaixo. Ele quer fazer uma única corrente de 30 elos. Paraabrir e depois soldar um elo o serralheiro leva 5 minutos. Quantos minutosno mínimo ele levará para fazer a corrente?

2001. Papa-Léguas participou de uma corrida (junto com o Ligeirinho eo Flash), que consistia em dar 100 voltas em um circuito. Como sempre, oCoiote queria pegar o Papa-Léguas e colocou um monte de alpiste no meio dapista. É claro que o Coiote não conseguiu pegar o Papa-Léguas, mas ele fezcom que a velocidade média dele na primeira volta fosse de apenas 200km/h.Sabendo disso, a velocidade média do Papa-Léguas na corrida:

A) Não ultrapassa 200km/h.B) Não ultrapassa 250km/h, mas pode ultrapassar 200km/h.C) Não ultrapassa 2000km/h, mas pode ultrapassar 250km/h.D) Não ultrapassa 20000km/h, mas pode ultrapassar os 2000km/h.E) Pode ultrapassar 20000km/h.

2002. Um comerciante comprou dois carros por um total de R$27.000, 00.Vendeu o primeiro com lucro de 10% e o segundo com prejuízo de 5%. Nototal ganhou R$750, 00. Quais foram os preços de compra?

2002. Os valores de x, y e z que satisfazem às equações x+ 1y

= 5, y+ 1z

= 1

e z + 1x

= 2. Quanto vale x+ 3y + 2z?

2002. Durante sua viagem ao país das Maravilhas a altura de Alice sofreuquatro mudanças sucessivas da seguinte forma: primeiro ela tomou um golede um líquido que estava numa garrafa em cujo rótulo se lia: "beba-me e�que 25% mais alta". A seguir, comeu um pedaço de uma torta onde es-tava escrito: "proveme e �que 10% mais baixa"; logo após tomou um goledo líquido de outra garrafa cujo rótulo estampava a mensagem: "beba-me e�que 10% mais alta". Finalmente, comeu um pedaço de outra torta na qualestava escrito:"prove-me e �que 20% mais baixa". Após a viagem qual era aaltura de Alice?

2002. A média aritmética das idades de um grupo de médicos e advogadosé 40 anos. A média aritmética das idades dos médicos é 35 anos e a dosadvogados é 50 anos. Pode-se, então, a�rmar que:

Page 40: Alan Anderson 30 de janeiro de 2017 · 6.4.10 Áreas para encontrar razões entre ... façam o mesmo com os outros níveis e ... Agora esse texto visa ajudar estudantes que não sabem

40 CAPÍTULO 3. ÁLGEBRA

A) O número de advogados é o dobro do número de médicos no grupo.B) O número de médicos é o dobro do número de advogados no grupo.C) Há um médico a mais no grupo.D) Há um advogado a mais no grupo.E) Existem as mesmas quantidades de médicos e advogados no grupo.

2002. Marcelo leva exatamente 20 minutos para ir de sua casa até a es-cola. Uma certa vez, durante o caminho, percebeu que esquecera em casa arevista Eureka! que ia mostrar para a classe; ele sabia que se continuasse aandar, chegaria à escola 8 minutos antes do sinal, mas se voltasse para pegara revista, no mesmo passo, chegaria atrasado 10 minutos. Que fração docaminho já tinha percorrido neste ponto?

2002.(FIGURA) A linha poligonal AB é desenhada mantendo-se sempreo mesmo padrão mostrado na �gura. Quanto vale o seu comprimento total ?

2003. Dois amigos, Augusto e Eduardo, atravessavam uma ponte ondepassava uma linha férrea. Quando tinham percorrido dois quintos da ponte,ouviram o barulho de um trem que se aproximava por trás deles. Apavora-dos, começaram a correr, cada um para o seu lado. Tiveram sorte: Augusto,que tinha voltado, conseguiu sair da ponte no exato instante em que o tremnela ia entrar. Por sua vez, Eduardo, que continuou para a frente, conseguiusair da ponte no instante em que o trem também ia fazê-lo. Refeitos do susto,quando se encontraram, comentaram que isto só foi possível porque correrama 15km/h e o trem estava a xkm/h. Qual é o valor de x?

3.9 Equações funcionais(Seção de exemplos)

3.9.1 Exercícios com funções

1998. A função f associa a cada real x o menor elemento do conjunto{x+ 1, 15−x

2

}. Qual é valor máximo de f(x)?

1999. Para todo n natural de�nimos a função f por: f(n) = n2se n é

par,f(n) = 3n + 1 se n é ímpar. Qual é número de soluções da equaçãof(f(f(n))) = 16.

Page 41: Alan Anderson 30 de janeiro de 2017 · 6.4.10 Áreas para encontrar razões entre ... façam o mesmo com os outros níveis e ... Agora esse texto visa ajudar estudantes que não sabem

3.9. EQUAÇÕES FUNCIONAIS(SEÇÃO DE EXEMPLOS) 41

2000. Seja f uma função real que tem as seguintes propriedades:i) Para todos x, y reais, f(x+ y) = x+ f(y);ii)f(0) = 2.Quanto vale f(2000)?

2001. Seja f(x) = x2 − 3x + 4. Quantas soluções reais tem a equaçãof(f(f(...f(x)))) = 2 (onde f é aplicada 2001 vezes)?

2001. Seja f uma função de Z em Z de�nida como f(x) = x/10 se x édivisível por 10 e f(x) = x+ 1 caso contrário. Se a0 = 2001 e an+1 = f(an),qual é o menor valor de n para o qual an = 1?

2002. Seja f uma função real de variável real que satisfaz a condição:

f(x) + f

(2002

x

)= 3x

para x > 0. Qual é o valor de f(2)?

2003. A função f é de�nida para todos os pares ordenados (x; y) deinteiros positivos e tem as seguintes propriedades: f(x;x) = x, f(x; y) =f(y;x), (x+ y)f(x; y) = (2x+ y)f(x;x+ y). Qual é o valor de f(21; 12)?

2003. Qual é o número de funções f : R → R tal que, para todox ∈ R, f(x)(f(x)− x) = 0?

2004. A função real f , de�nida nos inteiros, satisfaz f(n)− (n+ 1)f(2−n) = (n+ 3)2, para todo n inteiro. Quanto vale f(0)?

Page 42: Alan Anderson 30 de janeiro de 2017 · 6.4.10 Áreas para encontrar razões entre ... façam o mesmo com os outros níveis e ... Agora esse texto visa ajudar estudantes que não sabem

42 CAPÍTULO 3. ÁLGEBRA

Page 43: Alan Anderson 30 de janeiro de 2017 · 6.4.10 Áreas para encontrar razões entre ... façam o mesmo com os outros níveis e ... Agora esse texto visa ajudar estudantes que não sabem

Capítulo 4

Teoria dos Números

Pré-requisito: Indução.

4.1 Divisibilidade

Lema 1 (Euclides) Seja a um inteiro positivo. Dado qualquer inteiro bpodemos escrever de modo único b = aq + r com r ∈ {0, 1, ..., a− 1}.

Pela familiaridade que se tem com esse algoritmo, a prova será omitida,mas a�rmamos com muita convicção, que ela não é difícil, segue por indução.

Como já se sabe, o r no algoritmo anterior é chamado de resto da divisãode b por a.

Vejamos alguns exemplos:112 = 31× 3 + 19251 = 31× 8 + 81)Qual é resto da divisão de 112 + 251 = 363 por 31?

85 = 13× 6 + 7142 = 13× 10 + 122)Qual é o resto da divisão de 142− 85 = 57 por 13?

205 = 29× 7 + 2349 = 29× 12 + 1

43

Page 44: Alan Anderson 30 de janeiro de 2017 · 6.4.10 Áreas para encontrar razões entre ... façam o mesmo com os outros níveis e ... Agora esse texto visa ajudar estudantes que não sabem

44 CAPÍTULO 4. TEORIA DOS NÚMEROS

3)Qual é o resto da divisão de 205× 349 = 71545 por 29?

Estas perguntas estão aí para nos dar uma noção intuitiva do que iremosver em nosso objeto �nal. Começaremos com a seguinte

De�nição 6 Dizemos que b é múltiplo de a, a é divisor de b, ou que a divideb, e escrevemos a|b quando existe um inteiro c tal que b = ac. Caso contráriosescrevemos a 6 |b.

Em outros termos, dizer que a divide b signi�ca que baé inteiro. Ou ainda,

a divide b quando o resto da divisão de b por a é zero.

Exemplos:−13|26 pois existe um inteiro c, a saber c = −2, tal que 26 = −13c;1|n para qualquer inteiro n, pois n/1 = n e n que é um inteiro;17|51 pois o resto da divisão de 51 por 17 é igual a 0n|0, qualquer que seja n inteiro (diferente de 0), pois 0/n = 0 que é inteiro;8 6 |4 pois não existe um inteiro c tal que 4 = 8c;19 6 |189 pois 189/19 não é inteiro;14 6 |37 pois o resto da divisão de 37 por 14 é diferente de 0.

Exercícios

1. Encontre todos os pares de inteiros (x, y) tais que 13x2 + 39y = 2012.

2. Encontre todos os trios de inteiros (x, y, z) tais que 98xy + 343zx =70007.

3. Encontre o resto da divisão de 1 + 2 + 22 + ...+ 2100 por 4.

4. Mostre que 212 + 1|224 − 1.

5. Um inteiro a é dito livre de quadrados quando não existe nenhum inteirom 6= ±1 tal que m2|a. Qual é o maior número possível de inteirosconsecutivos livres de quadrados?

6. (IME) Mostre que (a− b)(a− c)(a− d)(b− c)(b− d)(c− d) é múltiplode 12, quaiquer que sejam a, b, c, d inteiros.

7. Quais são os possíveis resto da divisão de um quadrado perfeito por 4?

Page 45: Alan Anderson 30 de janeiro de 2017 · 6.4.10 Áreas para encontrar razões entre ... façam o mesmo com os outros níveis e ... Agora esse texto visa ajudar estudantes que não sabem

4.2. TEOREMA FUNDAMENTAL DA ARITMÉTICA 45

8. Encontre todos os trios de inteiros (x, y, z) tais que 4x2+y2+z2 = 3507.

Agora daremos algumas propriedades que facilitarão, no futuro, a resolu-ção de problemas.

Lema 2 i) Se d|a e d|b, então d|ax+ by para quaisquer x, y inteiros;ii) Se d|a, então a = 0 ou |a| ≥ |d|;iii) Se a|b e b|c então a|c.

Exemplo:Encontre todos os inteiros positivos n tais que 13n+ 20 divide 7n+ 59.

Solução: Como n é inteiro positivo 7n + 59 6= 0, assim devemos ter obri-gatoriamente |7n + 59| ≥ |13n + 20| ⇒ 7n + 59 ≥ 13n + 20 ⇒ 59 − 20 ≥13n−7n⇒ 39 ≥ 6n⇒ n ≤ 6, 5. Assim, temos seis possibilidades de inteirospositivos para testar. Ao testarmos veri�camos que a única solução é n = 1.

Exercícios

1. Encotre todos os inteiros positivos n, tais quea)2n− 1|n3 + 1;b)2n3 + 5|n4 + n+ 1;

4.2 Teorema Fundamental da Aritmética

Teorema 11 Sejam x, y ∈ Z e d = mdc(x, y), então existem a, b ∈ Z taisque

ax+ by = d.

De�nição 7 Dizemos que um número inteiro positivo p ≥ 2 é primo quandoos seus únicos divisores (positivos) são 1 e o próprio p.

Exemplos: 2, 3, 5, 7, 11, 13, 17, 19, 23, 29, 31, 37, 41, 43, 47, 53...

Lema 3 Se p é um número primo e p|ab então p|a ou p|b.

Teorema 1 (Teorema Fundamental da Aritmética) Seja n ≥ 2 um nú-mero natural. Podemos escrever n de uma única forma como o produton = p1p2...pm onde m ≥ 1 é um natural e p1 ≤ p2 ≤ ... ≤ pm são primos.

Page 46: Alan Anderson 30 de janeiro de 2017 · 6.4.10 Áreas para encontrar razões entre ... façam o mesmo com os outros níveis e ... Agora esse texto visa ajudar estudantes que não sabem

46 CAPÍTULO 4. TEORIA DOS NÚMEROS

Exemplos:360 = 2.2.2.3.3.514 = 2.798 = 2.7.7

2004. Se n! = 215 · 36 · 53 · 72 · 11 · 13, qual é o valor de n?Exercícios

1. Determine todos os inteiros positivos n tais que 2n−1 é divisível por 7.

2. Encontre todos os pares de inteiros positivos (x, y) tais que x2−y2 = 71.

3. Encontre todos os pares de inteiros positivos (x, y) tais que x2−y2 = 55.

4. Encontre todos os pares de inteiros positivos (x, y) tais que x2−y2 = 96.

5. Mostre que√

2 não é racional, isto é não existem p, q ∈ Z tais que√2 = p

q.

6. (Rioplatense) Prove que a equação q3 = 2n + 1 não tem solução nosinteiros positivos.

7. (OBM-2001)Mostre que não existem dois números inteiros a e b taisque (a+ b)(a2 + b2) = 2001.

4.3 Congruências

O uso de congruências é bastante comum na resolução de problemas de arit-mética e teoria dos números que surgem na jornada de um olímpico. Comesta técnica muitos problemas se tornam muito mais simples do que pare-cem. A congruêcia em geral, é apenas uma maneira simpli�cada de avaliardivisibilidade.

De�nição 8 Sejam a, b e n inteiros, dizemos que a é congruente a b módulon, e escrevemos a ≡ b(mod n), quando n|b− a.

Exemplos:7 ≡ 3(mod4) pois 4| − 4 = 3− 7−1 ≡ 1(mod2) pois 2|2 = 1− (−1)

Page 47: Alan Anderson 30 de janeiro de 2017 · 6.4.10 Áreas para encontrar razões entre ... façam o mesmo com os outros níveis e ... Agora esse texto visa ajudar estudantes que não sabem

4.4. EXERCÍCIOS DA OBM PRIMEIRA FASE 47

0 ≡ 65(mod− 13) pois −13|65 = 65− 0

Observações:1)Se a = nq + r é a decomposição obtida pelo algoritmo de Euclides, temosque r ≡ a(mod n), ou seja, o dividendo é congruente ao resto módulo odivisor, pois n|nq = a− r.2)Temos que a|b ⇐⇒ 0 ≡ b(mod a)

Agora pare! Volte ao início! Pense um pouco nas quatro perguntas que�zemos no início deste texto! Você consegue ver algo com outros olhos? Sevocê percebeu algo interessante você poderá �car feliz com a veracidade dosfatos que serão observados nas propriedades a seguir, fatos que tornarão oseu mundo bem mais simples!

4.4 Exercícios da OBM primeira fase

1998. Um fabricante de brinquedos embala bolas de pingue-pongue em doistipos de caixas. Num dos tipos ele coloca 10 bolas e no outro coloca 24 bo-las. Num certo dia foram embaladas 198 bolas e usadas mais de 10 caixas.Quantas caixas foram feitas nesse dia?

1998. O número 1234a6 é divisível por 7. Quanto valeo o algarismo a?

1998. Um número inteiro n é bom quando 4n + 1 é um múltiplo de 5.Quantos números bons há entre 500 e 1000?

1999. Quantos são os possíveis valores inteiros de x para quex+ 19

x+ 99seja

um número inteiro?

1999. O número N = 11111 · · · 11 possui 1999 dígitos, todos iguais a 1.Qual é resto da divisão de N por 7?

2001. O número N de três algarismos multiplicado por 7 deu como resul-tado um número que termina em 171. Determine N .

Page 48: Alan Anderson 30 de janeiro de 2017 · 6.4.10 Áreas para encontrar razões entre ... façam o mesmo com os outros níveis e ... Agora esse texto visa ajudar estudantes que não sabem

48 CAPÍTULO 4. TEORIA DOS NÚMEROS

2001. Os pontos P1, P2, P3, · · · estão nesta ordem sobre uma circunferên-cia e são tais que o arco que une cada ponto ao seguinte mede 35◦. Qual é omenor valor de n > 1 tal que Pn coincide com P1.

2001. Qual é o número de soluções inteiras distintas da equação

(−6x2 + 12x− 2)x2−2x+2 = 4?

2001. Quantos dígitos tem o menor quadrado perfeito cujos quatro últi-mos dígitos são 2001?

2001. São escritos todos os números de 1 a 999 nos quais o algarismo 1aparece exatamente 2 vezes (tais como, 11, 121, 411, etc). Qual é a soma detodos estes números?

2002. Qual é o dígito das unidades de 777...7

, onde aparecem 2002 setes?

2002. N = λ539984λ é um número inteiro positivo com oito algarismos,sendo o primeiro e o último desconhecidos. Sabendo que N é um múltiplode 198, encontre o algarismo das unidades de N/198.

2002. Sejam x, y, z números inteiros tais que x + y + z = 0. Sobrex3 + y3 + z3 quais a�rmativas seguintes são verdadeiras?

i) É necessariamente múltiplo de 2.ii) É necessariamente múltiplo de 3.iii) É necessariamente múltiplo de 5.

2002. Qual é o resto da divisão por 9 de√

1111111111− 22222?

2003. O número 19AB, onde A e B são dígitos, é um quadrado perfeito.Qual é o valor de

√AB da raiz quadrada do número cuja representação de-

cimal é AB?

2004. Para quantos inteiros positivos m o número

2004

m− 2

é um inteiro positivo?

Page 49: Alan Anderson 30 de janeiro de 2017 · 6.4.10 Áreas para encontrar razões entre ... façam o mesmo com os outros níveis e ... Agora esse texto visa ajudar estudantes que não sabem

4.5. PEQUENO FERMAT E GRANDE EULER 49

2004. Esmeralda, a digitadora, queria digitar um número N de dois al-garismos que é quadrado perfeito, mas se enganou, trocando cada algarismopelo seu sucessor (a�nal, as teclas são vizinhas!). Por uma grande coinci-dência, o número digitado também é quadrado perfeito! Qual é a soma dosalgarismos de N?

4.5 Pequeno Fermat e Grande Euler

Exercício Mostre que se p é primo, então para 1 ≤ k ≤ p − 1, temos quep|(pk

).

Teorema 12 (Pequeno Teorema de Fermat) Mostre que se p é primo,então ∀a ∈ N,

ap ≡ a (mod.p).

Prova: Mostre por indução em a usando o exercício acima.

De�nição 9 Dado um número natural n de�nimos φ(n) como sendo o aquantidade de números k ≤ n que são primos com n, ou seja, tais quemdc(n, k) = 1.

Obviamente φ é uma função de φ : N→ N. Esse é o primeiro exemplo defunção artitmética importante.

Exercícioa) Calcule φ(p) para p número primo.b) Calcule φ(p2) para p número primo.c) Calcule φ(pn) para p número primo e n ∈ N.

Teorema 13 (Euler) Seja n ∈ N, então ∀a ∈ N com mdc(a, n) = 1, temos

aφ(n) ≡ a (mod.n).

Observe que o teorema de Euler é uma generalização do Pequeno Teoremade Fermat. Mais precisamente, o teorema de Fermat segue do teorema deEuler no caso em que n é um número primo.

Page 50: Alan Anderson 30 de janeiro de 2017 · 6.4.10 Áreas para encontrar razões entre ... façam o mesmo com os outros níveis e ... Agora esse texto visa ajudar estudantes que não sabem

50 CAPÍTULO 4. TEORIA DOS NÚMEROS

4.6 Teorema Chinês dos Restos

Teorema 14 Se mdc(a, n) = 1, então existe x tal que a · x ≡ 1(mod.n).

ProblemaSejam a, b, c, d ∈ N dois a dois primos entre sia) Encontre x tal que x ≡ 0(mod.a), x ≡ 0(mod.b) e x ≡ 1(mod.c)b) Encontre x tal que x ≡ 0(mod.a), x ≡ 0(mod.b) e x ≡ d(mod.c).

Exercícioa) Encontre um número x tal que o resto da divisão de x por 12 é 0, por

13 é 0 e por 17 é é 8.b) Encontre um número y tal que o resto da divisão de y por 12 é 0, por

13 é 3 e por 17 é é 0.c) Encontre um número z tal que o resto da divisão de z por 12 é 9, por

13 é 0 e por 17 é é 0.d) Qual é o resto da divisão de x+y+z por cada um dos valores 12, 13, 17?

Teorema 15 Sejam m1,m2, ...,mr, inteiros positivos primos entre si, doisa dois, e sejam a1, a2, ..., ar; r inteiros quaisquer. Então, o sistema de con-gruências:

x ≡ a1(mod.m1)

x ≡ a2(mod.m2)

...

x ≡ ar(mod.mr)

admite uma solução x. Além disso, as soluções são únicas módulo m =m1m2...mr.

Referência: Aula 11, Poti Nível 2

Page 51: Alan Anderson 30 de janeiro de 2017 · 6.4.10 Áreas para encontrar razões entre ... façam o mesmo com os outros níveis e ... Agora esse texto visa ajudar estudantes que não sabem

Capítulo 5

Geometria

Pré-requisito: semelhança de triângulos, de�nição de seno e cosseno.

5.1 Fatos básicos

Um fato muito importante sobre triângulos é: se a soma de dois dos lados formenor que o terceiro o triângulo não fecha. Enunciaremos isso precisamenteabaixo.

Teorema 16 (Desigualdade Triangular) Sejam a, b, c os comprimentosdos lados de um triângulo. Então a+ b > c, a+ c > b e b+ c > a.

FIGURA2001. Para cada ponto pertencente ao interior e aos lados de um triân-

gulo acutângulo ABC, considere a soma de suas distâncias aos três lados dotriângulo. O valor máximo desta soma é igual A) à média aritmética das 3alturas do triângulo.

B) ao maior lado do triângulo.C) à maior altura do triângulo.D) ao triplo do raio do círculo inscrito no triângulo.E) ao diâmetro do círculo circunscrito ao triângulo.

Outro fato importante sobre triângulos diz respeito a soma dos seus ân-gulos internos.

Teorema 17 A soma dos ângulos internos de um triângulo é igual a 180o.

51

Page 52: Alan Anderson 30 de janeiro de 2017 · 6.4.10 Áreas para encontrar razões entre ... façam o mesmo com os outros níveis e ... Agora esse texto visa ajudar estudantes que não sabem

52 CAPÍTULO 5. GEOMETRIA

Prova: FIGURA.Se temos um polígono convexo, podemos fatiá-lo em pedaços triangulares

e usar o teorema acima repetidamente de modo a obter que

Teorema 18 A somas do ângulos internos de um polígono convexo com nlados é (n− 2)180o.

1998. Todos os ângulos internos de um polígono convexo são menores que(não podendo ser iguais a) 160o. Qual é o número máximo de lados dessepolígono?

2000. DEFG é um quadrado no exterior do pentágono regular ABCDE.Quanto mede o ângulo EAF?

2001. (FIGURA) O triângulo CDE pode ser obtido pela rotação do triân-gulo ABC de 90o no sentido anti-horário ao redor de C, conforme mostradono desenho abaixo. Quanto vale o ângulo α?

2001. ABCDE é um pentágono regular e ABF é um triângulo equiláterointerior. Quanto mede o ângulo FCD?

2004. Constrói-se o quadrado ABXY sobre o lado AB do heptágonoregular ABCDEFG, exteriormente ao heptágono. Determine a medida doângulo BXC, em radianos.

Teorema 19 Um polígono convexo com n lados possuin(n− 3)

2diagonais.

Prova: Exercício de contagem.

5.2 Pitágoras e Triângulo Retângulo

A seguir consideraremos, a menos que dito explicitamente o contrário, queABC é um triângulo retângulo com ∠A = α,∠B = 90o,∠C = β = 90o − α.Além disso H será o pé da altura relativa ao lado AC.

FIGURA

Page 53: Alan Anderson 30 de janeiro de 2017 · 6.4.10 Áreas para encontrar razões entre ... façam o mesmo com os outros níveis e ... Agora esse texto visa ajudar estudantes que não sabem

5.2. PITÁGORAS E TRIÂNGULO RETÂNGULO 53

Exercício: Mostre que sin(90− α) = cos(α).

Problema Mostre que AHAB

= ABAC

e CHCB

= CBCA

.

Teorema 20 (Pitágoras) Seja ABC um triângulo com ∠B = 90o, então

AB2 +BC2 = AC2.

Prova: Use o problema acima e prove o teorema de Pitágoras.

Exercícioa) Seja l o comprimento do lado de um quadrado, calcule o comprimento

da sua diagonal;b) Seja l o comprimento do lado de um triângulo equilátero, calcule sua

altura e sua área.OBM

1999. Um quadrado ABCD possui lado 40cm. Uma circunferência contémos vértices A e B e é tangente ao lado CD. Calcule o raio desta circunferência.

2000. (TEM DESENHO) A �gura abaixo mostra o logotipo de umaempresa, formado por dois círculos concêntricos e por quatro círculos demesmo raio, cada um deles tangente a dois dos outros e aos dois círculosconcêntricos. O raio do círculo interno mede 1 cm. Então o raio do círculoexterno deverá medir, em cm:

A) 2√

2 + 3 B)√

2 + 2 C) 4√

2 + 1 D) 3√

2 E)√

2 + 1

2001. (FIGURA) Uma mesa retangular, cujos pés têm rodas, deve serempurrada por um corredor de largura constante, que forma um ângulo reto.Se as dimensões da mesa são a e b (com 2a < b), qual deve ser a larguramínima do corredor para que a mesa possa ser empurrada através dele?

2003. A �gura abaixo mostra duas retas paralelas r e s. A reta r é tangenteàs circunferências C1 e C3, a reta s é tangente às circunferências C2 e C3 eas circunferências tocam-se como também mostra a �gura. (FIGURA)

As circunferências C1 e C2 têm raios a e b, respectivamente. Qual é oraio da circunferência C3?

Page 54: Alan Anderson 30 de janeiro de 2017 · 6.4.10 Áreas para encontrar razões entre ... façam o mesmo com os outros níveis e ... Agora esse texto visa ajudar estudantes que não sabem

54 CAPÍTULO 5. GEOMETRIA

2003. A �gura a seguir mostra um quadrado ABCD e um triânguloeqüilátero BEF, ambos com lado de medida 1cm . Os pontos A, B e E sãocolineares, assim como os pontos A, G e F. (FIGURA)

Qual é área do triângulo BFG é, em cm2?

2003. No triângulo ABC,AB = 20, AC = 21 e BC = 29. Os pontos De E sobre o lado BC são tais que BD = 8 e EC = 9. Qual é a medida doângulo DAE?

2004. (FIGURA) Dois espelhos formam um ângulo de 30o no ponto V .Um raio de luz, vindo de uma fonte S, é emitido paralelamente a um dosespelhos e é re�etido pelo outro espelho no ponto A, como mostra a �gura.Depois de uma certa quantidade de re�exões, o raio retorna a S. Se AS eAV têm 1 metro de comprimento, qual é a distância percorrida pelo raio deluz?

5.3 Informações básicas de trigonometria

Começamos essa seção com a informção base de trigonometria, sua impor-tância é expressa pelo seu título.

Teorema 21 (Relação Fundamental da Trigonometria) Seja ABC umtriângulo como acima, então

sin2 α + cos2 α = 1.

Prova: Use o teorema de pitágoras para mostrar isso.

5.3.1 Seno e Cosseno como funções reais

Seno, cosseno e tangente, podem ser vistas como funções reais ou funções dosgraus. A tradução de grau pra número real decorre da tradução de grau pararadianos. Trataremos de graus, se o leitor souber trabalhar com radiano,�ca como exercício fazer a tradução, se o leitor não souber trabalhar comradiano, �nja sugerimos que não esquente não se preocupe com isso durantea leitura, e vá aprender após a leitura.

Page 55: Alan Anderson 30 de janeiro de 2017 · 6.4.10 Áreas para encontrar razões entre ... façam o mesmo com os outros níveis e ... Agora esse texto visa ajudar estudantes que não sabem

5.3. INFORMAÇÕES BÁSICAS DE TRIGONOMETRIA 55

A seguir listaremos alguns fatos sobre tais funções (não iremos dar pro-vas).

A função seno

1. É uma função ímpar, ou seja, sin(−α) = − sinα;

2. É crescente no intervalo [0o, 90o] e decrescente no intervalo [90o, 180o].

A função cosseno

1. É uma função par, ou seja, cos(−α) = cosα;

2. É decrescente no intervalo [0o, 180o];

3. É injetiva no intervalo [0o, 180o] (isso decorre de 2);

4. Possui inversa no intervalo [0, 180o] (isso decorre de 3).

5.3.2 Ângulos notáveis

A seguir daremos alguns ângulos os quais são fáceis calcular os seus senose cossenos. Para determinar seus senos e cossenos. Para realizar esses cál-culos, utilizes o triângulo isóceles com ângulo maior de 90o e o uma dasmetades do triângulo equiláteros partido em dois por uma das suas altu-ras/medianas/bissetrizes.

sin 30o = 12, cos 30o =

√32

sin 45o =√22, cos 45o =

√32

sin 90o = 1

cos 180o = −1

Exercício Calcule sin 60o, cos 60o, cos 90o, sin 180o.

OBMF1-99 Se 0o < x < 90o e cosx = 14então x está entre:

A) 0o e 30o B)30o e 45o C) 45o e 60o D) 60o e 75o E) 75o e 90o

Page 56: Alan Anderson 30 de janeiro de 2017 · 6.4.10 Áreas para encontrar razões entre ... façam o mesmo com os outros níveis e ... Agora esse texto visa ajudar estudantes que não sabem

56 CAPÍTULO 5. GEOMETRIA

5.3.3 Fórmulas úteis

Teorema 22 (Seno da soma) Sejam α, β ângulos quaisquer, então valemas seguintes fórmulas

sin(α + β) = sinα cos β + sin β cosα

Prova: Essa demonstração será omitida.

Teorema 23 Sejam α, β ângulos quaisquer, então valem as seguintes fór-mulas

i) sin(α− β) = sinα cos β − sin β cosα

ii) cos(α + β) = cosα cos β − sin β sinα

iii) cos(α + β) = cosα cos β + sin β sinα.

Prova: Exercíco. Use a fórmula do seno da soma, e fatos como sin(−α) =sinα, sin 90o = 1, cos 180o = −1.

Exercício: Encontre fórmulas simples para sin(2α), cos(2α), sin(α2),

cos(α2) em termos de sinα e cosα.

Exercício: Calcule sin(180− α) e cos(180− α).

Teorema 24 Sejam α, β ângulos quaisquer, então valem as seguintes fór-mulas

i) sinα + sin β = 2 sin(α+β2

)cos(α−β2

);

ii) sinα− sin β = 2 sin(α−β2

)cos(α+β2

);

iii) cosα + cos β = 2 cos(α+β2

)cos(α−β2

);

iv) cosα− cos β = 2 sin(α+β2

)cos(α−β2

);

Prova: Use as fórmulas anteriores para provar isso.

Page 57: Alan Anderson 30 de janeiro de 2017 · 6.4.10 Áreas para encontrar razões entre ... façam o mesmo com os outros níveis e ... Agora esse texto visa ajudar estudantes que não sabem

5.4. LEI DOS COSSENOS 57

5.4 Lei dos Cossenos

Teorema 25 No triângulo ABC sejam a = BC, b = CA e c = AB. Então

a2 = b2 + c2 − 2bc cosA.

Claro que, análogamente temos b2 = c2 + a2 − 2ca cosB e c2 = a2 + b2 −2ab cosC.

A lei dos cossenos é uma recíproca generalizada do teorema de Pitágoras,pois ela diz que se o triângulo de lados a > b ≥ c não é retângulo (teremosque cosA 6= 0 e) então não vale a fórmula a2 = b2 + c2 (e diz muito mais,pois explica como calcular os lados em termos dos outros).

1999. Para quantos valores inteiros de x existe um triângulo acutângulode lados 12, 10 e x?

5.5 Ângulos Inscritos e Ângulos Tangentes

5.5.1 Ângulos inscritos

Lema 4 Dada uma circunferência Γ e um ponto P externo a ela, sejam r es as retas tangentes a Γ, em Q e R respectivamente, passando por P . EntãoPQ = PR.

Prova:Exercício: Considere o centro C de Γ e use o teorema de pitágoras. �

Teorema 26 (Identidades de Ravi) Sejam a, b, c os comprimentos dos la-dos de um triângulo. Então existem r, s, t > 0 tais que a = r+s, b = s+t, c =t+ r.

Prova: Use o Lema acima. �

Teorema 27 (Pitot) Se ABCD é um quadrilátero circunscrito a uma cir-cunferência, então AB + CD = AD +BC.

Prova: Use o Lema acima.�

2002. Na circunferência abaixo, temos que: AB = 4, BC = 2, AC édiâmetro e os ângulos ABD e CBD são iguais. Qual é o valor de BD?

Page 58: Alan Anderson 30 de janeiro de 2017 · 6.4.10 Áreas para encontrar razões entre ... façam o mesmo com os outros níveis e ... Agora esse texto visa ajudar estudantes que não sabem

58 CAPÍTULO 5. GEOMETRIA

5.5.2 Ângulos Tangentes

De�nição 10 O ângulo inscrito relativo a uma circunferência é um ânguloque tem o vértice na circunferência e os lados são secantes a ela.

Teorema 28 Um ângulo inscrito é metade do ângulo central correspondente.

Prova: Use segmento ligando o centro da circunferência ao vértice doângulo inscrito para provar isso.

Há muitas idéias com relação a esse aspecto, uma boa referência paraler mais sobre isso é ver a apostila do POTI que tem uma abordagem maiscompleta.

OBM1999. Os pontos S, T e U são os pontos de tangência do círculo inscrito

no triângulo PQR sobre os lados RQ,RP e PQ respectivamente. Sabendoque os comprimentos dos arcos TU, ST e US estão na razão TU : ST : US =5 : 8 : 11, qual é a razão ∠TPU : ∠SRT : ∠UQS?

2001. O hexágono ABCDEF é circunscritível. Se AB = 1, BC =2, CD = 3, DE = 4 e EF = 5, quanto mede FA?

5.6 Pontos, retas e segmentos notáveis

5.6.1 Bissetrizes

De�nição 11 A bissetriz de um ângulo é a semireta saindo do vértice quedivide o ângulo em duas parte iguais.

Teorema 29 Seja AD a bissetriz do ângulo ∠BAC no triâgulo ABC. Então

AB

BD=AC

CB.

Prova:

Teorema 30 O pontos da bissetriz são equidistantes dos lados que formamo ângulo bissectado.

Page 59: Alan Anderson 30 de janeiro de 2017 · 6.4.10 Áreas para encontrar razões entre ... façam o mesmo com os outros níveis e ... Agora esse texto visa ajudar estudantes que não sabem

5.6. PONTOS, RETAS E SEGMENTOS NOTÁVEIS 59

Prova: Exercício.

Teorema 31 As três bissetrizes dos ângulos de um triângulo se encontramem um único ponto interno do triângulo. Chamamos esse ponto de incentro.

Teorema 32 O incentro de um triângulo é equidistante dos seus três lados.

Assim, existe uma circunferência (e portanto um círculo) com centro em Ique tangencia os três lados do triângulo, ou seja, uma circunferência inscritaao triângulo. Chamamos o círculo de incírculo.

5.6.2 Medianas

De�nição 12 A mediana de um lado de um triângulo é o segmento que vaido vértice oposto a esse lado até o ponto médio do lado.

Teorema 33 As três medianas dos lados de um triângulo se encontram emum único ponto interno do triângulo. Chamamos esse ponto de baricentro.

Teorema 34 No triângulo ∆ABC sejam M e N os pontos médios de AB

e AC respectivamente. Então MN =BC

2. Além disso o triângulo formado

pelos pontos médios dos três lados é semelhante a ∆ABC

Prova: Semelhança de triângulos.

O segmento MN é chamado base média, às vezes é muito útil levá-laem consideração, portanto, quando vir pontos médios e não tiver ideia doque fazer, use a báse média.

5.6.3 Alturas

Teorema 35 As três alturas de um triângulo se encontram em um únicoponto H. Chamamos H de ortocentro do triângulo.

Se o triângulo é acutângulo, esse ponto é interno ao triângulo. Se otriângulo é retângulo, esse ponto é o vértice do ângulo reto. Se o triângulo éobtuso, esse ponto é externo ao triângulo.

Page 60: Alan Anderson 30 de janeiro de 2017 · 6.4.10 Áreas para encontrar razões entre ... façam o mesmo com os outros níveis e ... Agora esse texto visa ajudar estudantes que não sabem

60 CAPÍTULO 5. GEOMETRIA

5.6.4 Mediatrizes

Teorema 36 Os pontos da meditriz de um segmento são equidistantes dosextremos do segmento.

Prova: Exercício.

Teorema 37 As três mediatrizes de um triângulo se encontram em um únicoponto O. Chamamos O de circuncentro do triângulo.

Se o triângulo é acutângulo, esse ponto é interno ao triângulo. Se o triân-gulo é retângulo, esse ponto é o ponto médio da hipotenusa. Se o triânguloé obtuso, esse ponto é externo ao triângulo.

Teorema 38 O circuncentro é equidistante dos vértices do triângulo.

Assim, existe uma circunferência (e portanto um círculo) com centro emO e que possui três vértices do triângulo, ou seja, uma circunferência circuns-crita ao triângulo. Chamamos o círculo de circuncírculo.

5.7 Áreas de �guras planas

OBM

1999. (DESENHO) A circunferência abaixo tem raio 1, o arco AB mede70o e o arco BC mede 40o. Quanto mede área da região limitada pelas cordasAB e AC e pelo arco BC?

No quadrado ABCD o ponto E é médio de BC e o ponto F do lado CDé tal que o ângulo AEF é reto. Aproximadamente, que porcentagem a áreado triângulo AEF representa da área do quadrado?

A) 28% B) 31% C) 34% D) 36% E) 39%

2000. (FIGURA) Na �gura, as distâncias entre dois pontos horizontaisconsecutivos e as distâncias entre dois pontos verticais consecutivos são iguaisa 1. Qual é a área da região comum ao triângulo e ao quadrado?

Page 61: Alan Anderson 30 de janeiro de 2017 · 6.4.10 Áreas para encontrar razões entre ... façam o mesmo com os outros níveis e ... Agora esse texto visa ajudar estudantes que não sabem

5.7. ÁREAS DE FIGURAS PLANAS 61

2000. (FIGURA) Na �gura temos que os triângulos ABC e A?B?C? sãoequiláteros e a região destacada é um hexágono regular. Qual é a razão entrea área da região destacada e a área do triângulo ABC?

2002. Traçando segmentos, podemos dividir um quadrado em dois qua-dradinhos congruentes, quatro trapézios congruentes e dois triângulos con-gruentes, conforme indica o desenho abaixo, à esquerda. Eliminando algumasdessas partes, podemos montar o octógono representado à direita. Que fraçãoda área do quadrado foi eliminada?

5.7.1 Área do triângulo

1998. O quadrilátero ABCD é um quadrado de área 4m2. Os pontos M e Nestão no meio dos lados a que pertencem. Quanto vale a área do triânguloem destaque é, em m2? (ver �gura)

1999. (DESENHO) Dois irmãos herdaram o terreno ABC com a forma deum triângulo retângulo em A, e com o cateto AB de 84m de comprimento.Eles resolveram dividir o terreno em duas partes de mesma área, por ummuro MN paralelo a AC como mostra a �gura abaixo. Assinale a opção quecontém o valor mais aproximado do segmento BM.

A) 55m B) 57m C) 59m D) 61m E) 63m

2001. No triângulo ABC,AB = 5 e BC = 6. Qual é a área do triânguloABC, sabendo que o ângulo C tem a maior medida possível?

Notação:Dado um triângulo ∆ABC representaremos sua área por [ABC].Nos teoremas a seguir, a menos que dito expressamente o contrário, iremos

considerar que o triângulo tem lados com comprimentos AB = c, AC =b, BC = a.

Denotaremos o semiperímetro de ABC por s, ou seja, s =a+ b+ c

2.

Teorema 39 (Fórmula de Herão)

[ABC] =√s(s− a)(s− b)(s− c).

Page 62: Alan Anderson 30 de janeiro de 2017 · 6.4.10 Áreas para encontrar razões entre ... façam o mesmo com os outros níveis e ... Agora esse texto visa ajudar estudantes que não sabem

62 CAPÍTULO 5. GEOMETRIA

Teorema 40 Seja r o raio da circunferência inscrita em ABC.

[ABC] = s · r.

Prova: Some as áreas dos triâgulos ∆ABI, ∆ACI ∆CBI, onde I é oincentro do triângulo ABC.

Teorema 41[ABC] = a · b · sinC.

Prova: Mostre que b · sinC é a medida altura relativa ao lado BC.

Analogamente segue que [ABC] = b · c · sinA = c · a · sinB.

Teorema 42 (Lei dos Senos)

a

sinA=

b

sinB=

c

sinC.

Prova: Use o teorema acima.

Observação: As razões acima são também iguais a 2R, onde R é o raioda circunferência circunscrita ao triângulo.

5.8 Quadriláteros

De�nição 13 Um qudrilátero convexo é dito cíclico se pode ser inscrito numcirculo, ou seja, se seus vértices estão sobre um mesmo cículo.

Teorema 43 Se um quadrilátero é cíclico então a soma de dois ângulosopostos desse quadrilátero é 180o.

Prova: Exercício.

Teorema 44 (Ptolomeu) Se ABCD é um quadrilátero cíclico de diago-nais AC e BD, então:

AB × CD + AD ×BC = AC ×BD.

Teorema 45 (Brahmagupta) A medida da área de um quadrilátero cíclicode lados a, b, c, d cujo semiperímetro denotado por p é a seguinte:

K =√

(p− a)(p− b)(p− c)(p− d).

Page 63: Alan Anderson 30 de janeiro de 2017 · 6.4.10 Áreas para encontrar razões entre ... façam o mesmo com os outros níveis e ... Agora esse texto visa ajudar estudantes que não sabem

Capítulo 6

Enunciados dos Problemas

6.1 Combinatória e Probabilidade

6.1.1 classi�car

2001. Tem �gura O matemático excêntrico Jones, especialista em Teoriados Nós, tem uma bota com n pares de furos pelos quais o cadarço devepassar. Para não se aborrecer, ele gosta de diversi�car as maneiras de passaro cadarço pelos furos, obedecendo sempre às seguintes regras: ? o cadarçodeve formar um padrão simétrico em relação ao eixo vertical; ? o cadarçodeve passar exatamente uma vez por cada furo, sendo indiferente se ele ofaz por cima ou por baixo; ? o cadarço deve começar e terminar nos doisfuros superiores e deve ligar diretamente (isto é, sem passar por outros furos)os dois furos inferiores. Por exemplo, para n = 4, representamos a seguiralgumas possibilidades.

Determine, em função de n ? 2, o número total de maneiras de passaro cadarço pelos furos obedecendo às regras acima. Observação: Maneirascomo as exibidas a seguir devem ser consideradas iguais.

2002. Tem �guraColocamos vários palitos sobre uma mesa de modoa formar um retângulo m ? n, como mostra a �gura. Devemos pintar cadapalito de azul, vermelho ou preto de modo que cada um dos quadradinhosda �gura seja delimitado por exatamente dois palitos de uma cor e dois deoutra cor. De quantas formas podemos realizar esta pintura?

2014. Um conjunto é dito completamente divisível se para quaisquer

63

Page 64: Alan Anderson 30 de janeiro de 2017 · 6.4.10 Áreas para encontrar razões entre ... façam o mesmo com os outros níveis e ... Agora esse texto visa ajudar estudantes que não sabem

64 CAPÍTULO 6. ENUNCIADOS DOS PROBLEMAS

elementos a < b temos que a divide b. Um conjunto de inteiros positivos A écompletamente divisível e possui 2016 como um de seus elementos. Sabendoque todos os elementos de A são menores que 2 milhões, qual o máximo nú-mero de elementos que A pode ter?

2014. A mediana de um conjunto {a1, a2, ..., an} com a1 < a2 < ... < ané igual à media dos dois termos centrais (an

2+ an

2)/2 se n é par, e ao termo

central an+12

se n é ímpar. Sendo M a quantidade de subconjuntos de{1, 2, ..., 2014} com mediana igual a 2012 , encontre o resto da divisão deM por 2014.

6.1.2 Contagem

1998. Cinco cartões numerados com 3, 4, 5, 6 e 7, respectivamente, são co-locados em uma caixa. Os cartões são retirados da caixa, um de cada vez ecolocados sobre a mesa. Se o número de um cartão retirado é menor do que onúmero do cartão imediatamente anterior, então este cartão imediatamenteanterior é colocado de volta na caixa. O procedimento continua até que to-dos os cartões estejam sobre a mesa. Qual é o número máximo de vezes queretiramos cartões da caixa?

2001. Dizemos que um conjunto A formado por 4 algarismos distintos enão nulos é intercambiável se podemos formar dois pares de números, cadaum com 2 algarismos de A, de modo que o produto dos números de cadapar seja o mesmo e que, em cada par, todos os dígitos de A sejam utilizados.Por exemplo, o conjunto {1, 2, 3, 6} é intercambiável pois 21× 36 = 12× 63.Determine todos os conjuntos intercambiáveis.

2004. Os doze alunos de uma turma de olimpíada saíam para jogarfutebol todos os dias após a aula de matemática, formando dois times de6 jogadores cada e jogando entre si. A cada dia eles formavam dois timesdiferentes dos times formados em dias anteriores. Ao �nal do ano, eles veri�-caram que cada 5 alunos haviam jogado juntos num mesmo time exatamenteuma vez. Quantos times diferentes foram formados ao longo do ano?

2005. No campeonato tumboliano de futebol, cada vitória vale três pon-tos, cada empate vale um ponto e cada derrota vale zero ponto. Um re-

Page 65: Alan Anderson 30 de janeiro de 2017 · 6.4.10 Áreas para encontrar razões entre ... façam o mesmo com os outros níveis e ... Agora esse texto visa ajudar estudantes que não sabem

6.1. COMBINATÓRIA E PROBABILIDADE 65

sultado é uma vitória, empate ou derrota. Sabe-se que o Flameiras nãosofreu nenhuma derrota e tem 20 pontos, mas não se sabe quantas partidasesse time jogou. Quantas seqüências ordenadas de resultados o Flameiraspode ter obtido? Representando vitória por V , empate por E e derrotapor D, duas possibilidades, por exemplo, são (V,E,E, V, E, V, V, V, E,E) e(E, V, V, V, V, V, E, V ).

2006. O par ordenado (83, 89) é chamado de par centenário porque83 + 8 + 9 = 89 + 8 + 3 = 100, isto é, a soma de cada número com os dígitosdo outro número é 100. Quantos são os pares centenários?

2006. Seja n inteiro positivo. De quantas maneiras podemos distribuirn+ 1 brinquedos distintos para n crianças de modo que toda criança recebapelo menos um brinquedo?

2007. Um quadrado 4 × 4 é dividido em 16 quadrados unitários. Cadaum dos 25 vértices desses quadrados deve ser colorido de vermelho ou azul.Ache o número de colorações diferentes tais que cada quadrado unitário pos-sua exatamente dois vértices vermelhos.

2007. Em um certo país há 21 cidades e o governo pretende construirn estradas (todas de mão dupla), sendo que cada estrada liga exatamenteduas das cidades do país. Qual o menor valor de n para que, independentede como as estradas sejam construídas, seja possível viajar entre quaisquerduas cidades (passando, possivelmente, por cidades intermediárias)?

2008. Quantas permutações de 1, 2, 3, ..., 9 há com a propriedade de que,para todo 1 ≤ i ≤ 9, os números que aparecem entre i e i + 1 (onde i podeaparecer tanto antes como depois de i+ 1) são todos menores do que i? Porexemplo, 976412358 é uma permutação com esta propriedade.

2008. Determine a quantidade de funções f : {1, 2, 3, 4, 5} → {1, 2, 3, 4, 5}tais que f(f(x)) = f(x) para todo x ∈ {1, 2, 3, 4, 5}.

2009. Determine a quantidade de números n = a1a2a3a4a5a6, de seis al-garismos distintos, que podemos formar utilizando os algarismos 1, 2, 3, 4, 5, 6, 7, 8, 9de modo que as seguintes condições sejam satisfeitas simultaneamente:

i)a1 + a6 = a2 + a5 = a3 + a4;

Page 66: Alan Anderson 30 de janeiro de 2017 · 6.4.10 Áreas para encontrar razões entre ... façam o mesmo com os outros níveis e ... Agora esse texto visa ajudar estudantes que não sabem

66 CAPÍTULO 6. ENUNCIADOS DOS PROBLEMAS

ii)n é divisível por 9.

2010. Cada uma das oito casas de um retângulo de duas linhas e quatrocolunas é pintada de uma entre três cores. Uma coluna é chamada de cortese as suas duas casas são da mesma cor. De quantas maneiras é possívelpintar o retângulo de modo que haja exatamente um corte?

2010. Diamantino gosta de jogar futebol, mas se jogar dois dias segui-dos ele �ca com dores musculares. De quantas maneiras Diamantino podeescolher em quais de dez dias seguidos ele vai jogar bola sem ter dores mus-culares? Uma maneira é não jogar futebol em nenhum dos dias.

2011. Uma sequência de letras, com ou sem sentido, é dita alternadaquando é formada alternadamente por consoantes e vogais. Por exemplo,EZEQAF, MATEMÁTICA, LEGAL e ANIMADA são palavras alternadas,mas DSOIUF, DINHEIRO e ORDINÁRIO não são. Quantos anagramas dapalavra FELICIDADE (incluindo a palavra FELICIDADE) são sequênciasalternadas?

2011. Esmeralda tem um círculo de cartolina dividido em n setorescirculares, numerados de 1 a n, no sentido horário. De quantas maneirasEsmeralda pode pintar a cartolina, pintando cada setor com uma cor, tendodisponíveis k cores e de modo que quaisquer dois setores circulares vizinhos(isto é, que têm um segmento em comum como fronteira) tenham cores dife-rentes?

Note que isso implica que os setores de números 1 e n devem ter coresdiferentes.

2013. (FIGURA) Um bispo é uma peça do jogo de xadrez que só podefazer movimentos diagonais, isto é, ele pode se deslocar quantas casas quiserdesde que elas estejam em uma diagonal. Na �gura abaixo, indicamos aspossíveis direções de movimentos do bispo a partir de uma determinada casado tabuleiro. Dizemos que dois bispos se atacam quando um deles está emuma casa do tabuleiro que pode ser alcançada pelo outro bispo. Qual é omaior número de bispos que podemos colocar em um tabuleiro 8×8 sem quehaja dois bispos se atacando?

2013. Para cobrir um tabuleiro de dimensões 1× 112, podemos utilizar

Page 67: Alan Anderson 30 de janeiro de 2017 · 6.4.10 Áreas para encontrar razões entre ... façam o mesmo com os outros níveis e ... Agora esse texto visa ajudar estudantes que não sabem

6.1. COMBINATÓRIA E PROBABILIDADE 67

heptaminós amarelos, de dimensões 1× 7, e octaminós vermelhos, de dimen-sões 1× 8. De quantos modos podemos cobrir completamente o tabuleiro?

2014. Um conjunto é dito completamente divisível se para quaisquer ele-mentos a < b temos que a divide b . Um conjunto de inteiros positivos A écompletamente divisível e possui2016 como um de seus elementos. Sabendoque todos os elementos de A são menores que 2 milhões, qual o máximo nú-mero de elementos que A pode ter?

2015. Um subconjunto de 5 elementos do conjunto {1, 2, 3, ..., 20} é ditolargo se ao colocar os seus elementos em ordem crescente tivermos a propri-edade de que a diferença do segundo menos o primeiro é maior que 1, doterceiro para o segundo é maior que 2, do quarto para o terceiro é maior que3 e do quinto para o quarto é maior que 4. Existem quantos subconjuntoslargos?

6.1.3 Jogos e Invariantes

1998. Existem 20 balas sobre uma mesa e duas crianças começam a comê-las, uma criança de cada vez. Em cada vez, cada criança deve comer pelomenos uma bala e está proibida de comer mais que a metade das balas queexistem sobre a mesa. Nesta brincadeira, ganha a criança que deixar apenasuma bala sobre a mesa. Qual das duas crianças pode sempre ganhar na brin-cadeira: a primeira ou a segunda a jogar? Como deve fazer para ganhar?

1999. Nos extremos de um diâmetro de um círculo, escreve-se o número1 (primeiro passo) . A seguir, cada semicírculo é dividido ao meio e em cadaum dos seus pontos médios escreve-se a soma dos números que estão nosextremos do semicírculo (segundo passo) . A seguir, cada quarto de círculo édividido ao meio e em cada um dos seus pontos médios coloca-se a soma dosnúmeros que estão nos extremos de cada arco (terceiro passo). Procede-se,assim, sucessivamente: sempre cada arco é dividido ao meio e em seu pontomédio é escrita a soma dos números que estão em seus extremos. Determinara soma de todos os números escritos após 1999 passos.

1999. Determine todos os inteiros positivos n para os quais é possívelmontarmos um retângulo 9× 10 usando peças 1× n.

Page 68: Alan Anderson 30 de janeiro de 2017 · 6.4.10 Áreas para encontrar razões entre ... façam o mesmo com os outros níveis e ... Agora esse texto visa ajudar estudantes que não sabem

68 CAPÍTULO 6. ENUNCIADOS DOS PROBLEMAS

2000. O campeonato Venusiano de futebol é disputado por 10 times, emdois turnos. Em cada turno cada equipe joga uma vez contra cada uma dasoutras. Suponha que o Vulcano FC vença todas as partidas do 1o. turno.Caso não vença o 2o. turno, o Vulcano FC jogará uma �nal contra o vencedordo 2o turno, na qual terá vantagem caso faça mais pontos que o adversáriodurante todo o campeonato (vitória vale 3 pontos, empate vale 1 ponto ederrota 0 pontos).

a) Determine o menor n tal que, se o Vulcano FC �zer exatamente npontos no segundo turno, garantirá pelo menos a vantagem na �nal (inde-pendente de contra quem e com que placares conquiste os n pontos).

b) Determine o menor n tal que, se o Vulcano FC �zer pelo menos npontos no segundo turno, garantirá pelo menos a vantagem na �nal (inde-pendente de contra quem e com que placares conquiste os n pontos).

2000.Para efetuar um sorteio entre os n alunos de uma escola (n > 1)se adota o seguinte procedimento. Os alunos são colocados em roda e inicia-se uma contagem da forma "um, DOIS, um, DOIS,...". Cada vez que sediz DOIS o aluno correspondente é eliminado e sai da roda. A contagemprossegue até que sobre um único aluno, que é o escolhido.

a) Para que valores de n o aluno escolhido é aquele por quem começou osorteio?

b) Se há 192 alunos na roda inicial, qual é a posição na roda do alunoescolhido?

2012. Esmeralda e Jade, secretárias da OBM, jogam Destrua os triân-gulos. Esse jogo é disputado da seguinte forma: tem-se uma esfera e 2012pontos sobre a esfera. Em princípio todos os pares de pontos estão ligadospor um segmento. Esmeralda e Jade apagam, alternadamente, um segmento.A secretária que eliminar o último triângulo da esfera vence o jogo. Note quepodem sobrar segmentos no �nal do jogo; eles só não formam triângulo. SeEsmeralda começa o jogo, qual das secretárias tem estratégia vencedora, ouseja, vence o jogo não importando como o oponente jogue? Justi�que suaresposta, exibindo uma estratégia que funcione sempre.

2015. Considere um tabuleiro 2015× 37, pintado como um tabuleiro de

Page 69: Alan Anderson 30 de janeiro de 2017 · 6.4.10 Áreas para encontrar razões entre ... façam o mesmo com os outros níveis e ... Agora esse texto visa ajudar estudantes que não sabem

6.1. COMBINATÓRIA E PROBABILIDADE 69

xadrez. Cada linha e coluna tem um botão que inverte a cor de cada casinhada linha ou coluna correspondente, num total de 2015 + 37 = 2052 botões.Quantas colorações diferentes do tabuleiro podem ser obtidas?

6.1.4 Probabilidade

1999. José tem três pares de óculos, um magenta, um amarelo e um ciano.Todo dia de manhã ele escolhe um ao acaso, tendo apenas o cuidado de nuncausar o mesmo que usou no dia anterior. Se dia primeiro de agosto ele usouo magenta, qual a probabilidade de que dia 31 de agosto ele volte a usar omagenta?

2002. Quantos dados devem ser lançados ao mesmo tempo para maxi-mizar a probabilidade de se obter exatamente um 2?

2003. Um quadrado de lado 3 é dividido em 9 quadrados de lado unitá-rio, formando um quadriculado. Cada quadrado unitário é pintado de azulou vermelho. Cada cor tem probabilidade 1

2de ser escolhida e a cor de cada

quadrado é escolhida independentemente das demais. Qual a probabilidadede obtermos, após colorirmos todos os quadrados unitários, um quadrado delado 2 pintado inteiramente de uma mesma cor?

2006. Ao jogarmos uma certa quantidade de dados cúbicos com facesnumeradas de 1 a 6, a probabilidade de obtermos soma dos pontos 2006 éigual à probabilidade de obtermos soma dos pontos S. Qual é o menor valorpossível de S?

2007. Considere o conjunto A dos pares ordenados (x, y) de reais nãonegativos tais que x+ y = 2. Se a probabilidade de um elemento de A esco-lhido aleatoriamente estar a uma distância da origem menor ou igual a 5

p, quanto vale 2535p2?

2009. No programa de auditório Toto Bola, o apresentador Ciço Magal-lanes dispõe de duas caixas idênticas. Um voluntário da platéia é chamadoa participar da seguinte brincadeira: ele recebe dez bolas verdes e dez bolasvermelhas e as distribui nas duas caixas, sem que o apresentador veja, e demodo que em cada caixa haja pelo menos uma bola. Em seguida, o apre-

Page 70: Alan Anderson 30 de janeiro de 2017 · 6.4.10 Áreas para encontrar razões entre ... façam o mesmo com os outros níveis e ... Agora esse texto visa ajudar estudantes que não sabem

70 CAPÍTULO 6. ENUNCIADOS DOS PROBLEMAS

sentador escolhe uma das caixas e retira uma bola. Se a bola for VERDE,o voluntário ganha um carro. Se for VERMELHA, ele ganha uma banana.A máxima probabilidade que o voluntário tem de ganhar um carro é igual amn, em que m e n são inteiros positivos primos entre si. Determine o valor de

m+ n.

2015. . Três pontos A,B e C são marcados no bordo de um círculo demodo que m(BAC) = 60o, m(ABC) = 80o, m(ACB) = 40o. Escolhemos aoacaso um ponto X no interior do círculo. A probabilidade de que, entre ospontos A,B e C, o mais distante de X seja B é p

q, em que p e q são primos

entre si. Quanto vale p · q?

6.2 Álgebra

6.2.1 Funcões e Recorrências

1998. Seja f : N → R uma função tal que f(1) = 999 e f(1) + f(2) + ... +f(n) = n2 · f(n), para todo inteiro positivo n. Determine o valor def(1998).

2001. Determine todas as funções f : R → R tais que f(x) = f(−x) ef(x+ y) = f(x) + f(y) + 8xy + 115 para todos os reais x e y.

2001. Seja f(x) =x2

1 + x2. Calcule

f

(1

1

)+ f

(2

1

)+ f

(3

1

)+ ...+ f

(n1

)f

(1

2

)+ f

(2

2

)+ f

(3

2

)+ ...+ f

(n2

)f

(1

3

)+ f

(2

3

)+ f

(3

3

)+ ...+ f

(n3

)...

f

(1

n

)+ f

(2

n

)+ f

(3

n

)+ ...+ f

(nn

)2002. O primeiro número de uma seqüência é 7. O próximo é obtido

da seguinte maneira: Calculamos o quadrado do número anterior (72 = 49)

Page 71: Alan Anderson 30 de janeiro de 2017 · 6.4.10 Áreas para encontrar razões entre ... façam o mesmo com os outros níveis e ... Agora esse texto visa ajudar estudantes que não sabem

6.2. ÁLGEBRA 71

e a seguir efetuamos a soma de seus algarismos e adicionamos 1, isto é, osegundo número é 4 + 9 + 1 = 14. Repetimos este processo, e de 142 = 196,temos que o terceiro número da seqüência é 1 + 9 + 6 + 1 = 17, e assimsucessivamente. Qual o 2002o elemento desta seqüência?

2005. Determine o menor valor possível do maior termo de uma progres-são aritmética com todos os seus sete termos a1, a2, a3, a4, a5, a6, a7 primospositivos distintos.

2005. A função f : R→ R satisfaz f(x+ f(y)) = x+ f(f(y)) para todosos números reais x e y. Sabendo que f(2) = 8 , calcule f(2005).

2006. A seqüência Fn é de�nida por F1 = F2 = 1 e Fn = Fn−1 + Fn−2para n ≥ 3. Encontre todos os pares de inteiros positivos (m,n) tais queFm · Fn = mn.

2011. Seja f uma função dos reais não nulos nos reais não nulos tal que

i)(f(x) + f(y) + f(z))2 = (f(x))2 + (f(y))2 + (f(z))2 para todos x, y, zreais não nulos tais que x+ y + z = 0;

ii)f(−x) = −f(x) para todo x real não nulo;

iii)f(2011) = 1.

Encontre o inteiro mais próximo de f(33).

2014 A sequência a1, a2, a3, ... satisfaz a1 = 1 e an =√a2n−1 + n . Qual

é o inteiro mais próximo de a2014?

2015. Sejam f e g funções dos inteiros não negativos nos inteiros nãonegativos tais que f(0) = g(0) = 0, f(2x + 1) = g(x), g(2x) = f(x) ef(2x) = g(2x+ 1) = x para todo x inteiro não negativo. Quantos valores den tais que 0 ≤ n ≤ 2015 satisfazem f(n) = 0?

Page 72: Alan Anderson 30 de janeiro de 2017 · 6.4.10 Áreas para encontrar razões entre ... façam o mesmo com os outros níveis e ... Agora esse texto visa ajudar estudantes que não sabem

72 CAPÍTULO 6. ENUNCIADOS DOS PROBLEMAS

6.2.2 Somatórios e Produtórios

2000. O número√

1 + 112

+ 122

+√

1 + 122

+ 132

+ ... +√

1 + 120002

+ 120012

éracional; escreva-o na forma p

q, p e q inteiros.

2003. Calcule a soman∑k=0

2k+1

32k + 1.

2004. Cada um dos números x1, x2, ..., x2004 pode ser igual a√

2− 1 ou a√

2+1 . Quantos valores inteiros distintos a soman∑k=0

x2k−1x2k pode assumir?

2010. Calcule(24 + 22 + 1)(44 + 42 + 1)(64 + 62 + 1)...(324 + 322 + 1)

(14 + 12 + 1)(34 + 32 + 1)(54 + 52 + 1)...(314 + 312 + 1).

2013. Observe que 1n(n+1)

= 1n− 1

n+1. Assim podemos calcular a série

∞∑n=1

1

n(n+ 1)=

1

1 · 2+

1

2 · 3+

1

3 · 4+... =

(1− 1

2

)+

(1

2− 1

3

)+

(1

3− 1

4

)+... = 1.

Sabendo que∞∑n=1

1

n2=π2

6, o valor de

∞∑n=1

1

n(n+ 1)2é da forma A− π2

Bcom

A e B inteiros positivos. Determine o valor de A+B.

6.2.3 Equações do segundo grau e Polinômios

2005. Você tem que determinar o polinômio p(x) de coe�cientes inteirospositivos fazendo perguntas da forma "Qual é o valor numérico de p(k)?",sendo k um inteiro positivo à sua escolha.

Qual é o menor número de perguntas su�ciente para garantir que se des-cubra o polinômio?

2008. Suponha que a α ∈ R é raiz de algum polinômio não-nulo comcoe�cientes racionais. O polinômio minimal de α é o polinômio de menorgrau m(x) tal que:

i)m(a) = 0;ii)m(x) é Mônico (isto é, o seu coe�ciente líder é 1) e todos os seus

coe�cientes são racionais.Por exemplo, o polinômio minimal de

√2 é x2 − 2.

Page 73: Alan Anderson 30 de janeiro de 2017 · 6.4.10 Áreas para encontrar razões entre ... façam o mesmo com os outros níveis e ... Agora esse texto visa ajudar estudantes que não sabem

6.2. ÁLGEBRA 73

Determine o produto dos coe�cientes não nulos do polinômio minimal de3√−27 + 5

√33− 3

√27 + 5

√33.

2010. Sejam r e s números inteiros. Sabe-se que a equação do segundograu x2 − (r + s)x + rs + 2010 = 0 tem as duas soluções inteiras. Quantossão os possíveis valores de |r − s|?

2011. A equação do segundo grau x2 − 5x + m = 2011 tem pelo me-nos uma solução inteira. Qual é o menor valor inteiro positivo possível de m?

2011. Seja P (x) um polinômio de coe�cientes inteiros. Sabe-se queP (x) = 2011 tem pelo menos duas raízes inteiras distintas iguais a 1 e t, eque P (x) = 0 tem pelo menos uma raiz inteira. Determine todos os possíveisvalores de t.

2012. Considere a equação ax2 + bx + c = 0, em que a, b e c são reaise a > 0. Suponha que esta equação tenha duas raízes reais r e s tais que0 < r < 1 e 0 < s < 1. Mostre que b+ c < 0.

2013. Um retângulo, o qual não é um quadrado, tem lados com compri-mentos inteiros, medidos em centímetros. Se o seu perímetro é n centímetrose sua área é n centímetros quadrados, determine n.

2015. O professor Piraldo passou para Esmeralda uma equação da formaax = b, sendo a e b reais. Esmeralda se enganou e resolveu a equação bx = a,obtendo uma solução que é igual à correta menos 60. Se a solução correta éda forma m+

√n com m e n inteiros, qual é o valor de m+ n?

6.2.4 Parte Inteira e Aproximação

2007. Qual é a soma dos algarismos do inteiro mais próximo de√

111...111,(1000 uns)?

2008. Calcule o valor da soma

b 4√

1c+ b 4√

2c+ ...+ b 4√

2008c.

Page 74: Alan Anderson 30 de janeiro de 2017 · 6.4.10 Áreas para encontrar razões entre ... façam o mesmo com os outros níveis e ... Agora esse texto visa ajudar estudantes que não sabem

74 CAPÍTULO 6. ENUNCIADOS DOS PROBLEMAS

2009. Para cada inteiro positivo n, seja An = {x ∈ R+; bxc = n}, emque R+ é o conjunto dos reais positivos e bxc é o maior inteiro menor ouigual a x. Determine a quantidade de elementos do conjunto

A1 ∪ A2 ∪ ... ∪ A2009.

6.2.5 Cotas e desigualdades

2008. Em uma matriz 2008 × 2008 o elemento na linha i e coluna j é onúmero i + j (as linhas e colunas são numeradas de 1 a 2008). Escolhem-se2008 elementos desta matriz de modo que não haja dois elementos escolhidosnuma mesma linha ou coluna. Os elementos são multiplicados. Qual o menorproduto que se pode obter desta forma?

2009. Seja c a maior constante real para a qual

x2 + 3y2 ≥ c(x2 + xy + 4y2),

para todos x, y reais. Determine o inteiro mais próximo de 2009c.

6.2.6 Várias variáveis

2006. Esmeralda e Jade correm em sentidos opostos em uma pista circular,começando em pontos diametralmente opostos. O primeiro cruzamento en-tre elas ocorre depois de Esmeralda ter percorrido 200 metros. O segundocruzamento ocorre após Jade ter percorrido 350 metros entre o primeiro e osegundo ponto de encontro. As velocidades das moças são constantes. Qualé o tamanho da pista, em metros?

2010. Encontre todas as soluções inteiras não negativas x ≤ y ≤ z dosistema {

x+ y + z = 77xy + yz + zx+ xyz = 946

2011. Encontre todas as soluções reais (x, y, z) do sistema2y = x+ 1

x

2z = y + 1y

2x = z + 1z

Page 75: Alan Anderson 30 de janeiro de 2017 · 6.4.10 Áreas para encontrar razões entre ... façam o mesmo com os outros níveis e ... Agora esse texto visa ajudar estudantes que não sabem

6.3. TEORIA DOS NÚMEROS 75

2012. Sendo a, b, c reais tais que ab(a+b+c) = 1001, bc(a+b+c) = 2002e ca(a+ b+ c) = 3003, encontre abc.

2013. Determine o número de quádruplas (x, y, z, w) de reais tais quex3 = y + z + wy3 = z + w + xz3 = w + x+ yw3 = x+ y + z

2015. Os reais a, b e c satisfazem as equações

1

ab= b+ 2c,

1

bc= 2c+ 3a,

1

ca= 3a+ b.

Temos (a+b+c)3 = pq, com p e q inteiros primos entre si e q > 0. Calcule p+q.

6.3 Teoria dos Números

6.3.1 Equações e divisibilidade

1998. O menor múltiplo de 1998 que possui apenas os algarismos 0 e 9 é9990. Qual é o menor múltiplo de 1998 que possui apenas os algarismos 0 e 3?

1999. Encontre as soluções inteiras de x3 − y3 = 999.

1999. Determine o maior natural n para o qual existe uma reordena-ção (a, b, c, d) de (3, 6, 9, 12) (isto é, a, b, c, d = 3, 6, 9, 12) tal que o númeron√

3a6b9c12d seja inteiro. Justi�que sua resposta.

2001. Se a n-ésima OBM é realizada em um ano que é divisível por n,dizemos que esse ano é super-olímpico. Por exemplo, o ano 2001, em que estásendo realizada a 23a OBM, é super-olímpico pois 2001 = 87× 23 é divisívelpor 23. Determine todos os anos super-olímpicos, sabendo que a OBM nuncadeixou de ser realizada desde sua primeira edição, em 1979, e supondo quecontinuará sendo realizada todo ano.

Page 76: Alan Anderson 30 de janeiro de 2017 · 6.4.10 Áreas para encontrar razões entre ... façam o mesmo com os outros níveis e ... Agora esse texto visa ajudar estudantes que não sabem

76 CAPÍTULO 6. ENUNCIADOS DOS PROBLEMAS

2002. Determine o maior natural k para o qual existe um inteiro n talque 3k divide n3 − 3n2 + 22.

2003. Dizemos que um número N de quatro algarismos é biquadradoquando é igual à soma dos quadrados de dois números: um é formado pelosdois primeiros algarismos de N , na ordem em que aparecem em N e o outro,pelos dois últimos algarismos de N , também na ordem em queaparecem emN .

Por exemplo, 1233 é biquadrado pois 1233 = 122 + 332. Encontre umoutro número biquadrado.

2004. Determine todas as soluções da equação n · 2n−1 = m2 − 1 com ne m naturais.

2005. Determine todos os pares de inteiros (x, y) tais que

9xy − x2 − 8y2 = 2005.

2006. Encontre todos os pares de inteiros positivos (a, b) tais que ab+ 1divide (a+ 1)(b+ 1).

2007. Ache todos os pares (x, y) de inteiros positivos tais que

2(x+ y) + xy = x2 + y2.

2007. Encontre todos os números n de seis algarismos da formaAAABBB,em que A e B são algarismos diferentes e não nulos e n + 1 é um quadradoperfeito.

2008. Um inteiro positivo n é chamado de auto-replicante se os últimosdígitos de n2 formam o número n. Por exemplo, 25 é auto-replicante pois252 = 625. Determine a soma de todos os números auto-replicantes comexatamente 4 dígitos .

2008. Determine todos os inteiros positivosm e n tais quem2+161 = 3n.

2010. Seja N o menor número inteiro positivo que multiplicado por 33resulta em um número cujos algarismos são todos iguais a 7. Determine a

Page 77: Alan Anderson 30 de janeiro de 2017 · 6.4.10 Áreas para encontrar razões entre ... façam o mesmo com os outros níveis e ... Agora esse texto visa ajudar estudantes que não sabem

6.3. TEORIA DOS NÚMEROS 77

soma dos algarismos de N .

2011. Qual é o maior valor possível do mdc de dois números distintospertencentes ao conjunto {1, 2, 3, ..., 2011}?

2012. Os dois menores números primos da forma n2 + 5 são 62 + 5 = 41e 122 + 5 = 149. Qual é o terceiro menor primo dessa forma?

2013. Seja A = {1, 2, ..., 20} o conjunto dos 20 primeiros inteiros positi-vos. Para cada subconjunto X de 15 elementos de a , calculamos o produtop(X) de seus elementos. Por exemplo, p({1, 2, ..., 15}) = 1 · 2 · ... · 15 = 15!.Qual é o máximo divisor comum dos

(2015

)produtos p(X) obtidos com todos

os subconjuntos de 15 elementos de A?

2013. Escrevemos a soma dos recíprocos dos números de a como a fraçãoirredutível , ou seja,

1 +1

2+

1

3+ ...+

1

2013=A

B, mdc(A,B) = 1.

Qual é o maior valor inteiro n tal que B é múltiplo de 3n.

6.3.2 Teorema Fundamental da Aritimética

2000. Qual é o menor inteiro positivo que é o dobro de um cubo e o quín-tuplo de um quadrado?

2004. Dizemos que um número inteiro positivo é sinistro quando a somade seus fatores primos é igual à soma dos expoentes de sua decomposição emfatores primos. Encontre todos os números sinistros de quatro algarismos.

2007. Quantos divisores positivos do número 123456 são menores que2007?

2012. Arnaldo pensou em um número de quatro dígitos e desa�ou Ber-nardo a descobrir qual era o número. Para tanto, passou as seguintes trêsdicas para Bernardo, sendo que exatamente uma das dicas é falsa.

i) Dica 1: O número é um cubo perfeito;

Page 78: Alan Anderson 30 de janeiro de 2017 · 6.4.10 Áreas para encontrar razões entre ... façam o mesmo com os outros níveis e ... Agora esse texto visa ajudar estudantes que não sabem

78 CAPÍTULO 6. ENUNCIADOS DOS PROBLEMAS

ii)Dica 2: O número é o menor número de quatro dígitos que possui quatrodivisores positivos;

iii) Dica 3: O número é múltiplo de 59.Qual o número pensado por Arnaldo?

6.3.3 Teorema Chinês dos Restos e Sistemas de Con-gruências

2005. Seja a um número inteiro positivo tal que a é múltiplo de 5, a + 1 émúltiplo de 7, a + 2 é múltiplo de 9 e a + 3 é múltiplo de 11. Determine omenor valor que a pode assumir.

2009. (�gura) Sejam m e n dois inteiros positivos primos entre si. OTeorema Chinês dos Restos a�rma que, dados inteiros i e j com 0 ≤ i < me 0 ≤ j < n, existe exatamente um inteiro a, com 0 ≤ a < m · n, tal queo resto da divisão de a por m é igual a i e o resto da divisão de a por n éigual a j. Por exemplo, para m = 3 e n = 7, temos que 19 é o único númeroque deixa restos 1 e 5 quando dividido por 3 e 7, respectivamente. Assim,na tabela a seguir, cada número de 0 a 20 aparecerá exatamente uma vez.

Qual a soma dos números das casas destacadas?

2009. Determine o maior inteiro n menor que 10000 tal que 2n + n sejadivisível por 5.

2014. . O imparial de n é igual ao produto de todos os naturais ímparesmenores ou iguais a n. Quais são os três últimos algarismos do imparial de2014?

6.3.4 Fermat, Euler e Wilson

2015. Qual é o menor inteiro a > 1 para o qual existe n inteiro positivo talque a2

n − 1 é múltiplo de 2015?

Page 79: Alan Anderson 30 de janeiro de 2017 · 6.4.10 Áreas para encontrar razões entre ... façam o mesmo com os outros níveis e ... Agora esse texto visa ajudar estudantes que não sabem

6.4. GEOMETRIA 79

6.4 Geometria

6.4.1 Classi�car

1998. Uma reta que passa pelos pontos médios de dois lados opostos de umquadrilátero convexo forma ângulos iguais com ambas as diagonais. Mostreque as duas diagonais têm o mesmo comprimento.

2000.(tem �gura) O retângulo ao lado está dividido em 9 quadrados,A, B, C, D, E, F, G, H e I. O quadrado A tem lado 1. Qual é o lado doquadrado I?

2002. Para quais inteiros positivos n existe um polígono não regular den lados, inscrito em uma circunferência, e com todos os ângulos internos demesma medida?

2005.(�gura) Um prisma é reto e tem como base um triângulo equi-látero. Um plano corta o prisma mas não corta nenhuma de suas bases,determinando uma secção triangular de lados a, b e c. Calcule o lado da basedo prisma em função de a, b e c.

2006.(�gura) Na �gura a seguir, o pentágono regular ABCDE e o tri-ângulo EFG estão inscritos na circunferência Co, e M é ponto médio de BC.Para qual valor de α , em graus, os triângulos EFG e HIG são semelhantes?

2006. Qual a maior quantidade de lados que pode ter uma secção deter-minada por um plano em um octaedro regular?

2008. Um trapézio ABCD, com lados paralelos AB e CD, está inscritoem uma circunferência de raio 25. Sabe-se que CD é um diâmetro e a alturadesse trapézio é 24. Seja E um ponto no arco menor determinado por A e B esejam F e G os pontos de interseção de ED e EC com AB, respectivamente.

CalculeAF ·BGFG

.

2009. No triângulo ABC, temos ∠A = 120◦ e BC = 12cm. A circunfe-rência inscrita em ABC tangencia os lados AB e AC, respectivamente, nospontos D e E. Sejam K e L os pontos onde a reta DE intersecta a circun-ferência de diâmetro BC. Determine a distância entre os pontos médios dos

Page 80: Alan Anderson 30 de janeiro de 2017 · 6.4.10 Áreas para encontrar razões entre ... façam o mesmo com os outros níveis e ... Agora esse texto visa ajudar estudantes que não sabem

80 CAPÍTULO 6. ENUNCIADOS DOS PROBLEMAS

segmentos BC e KL.

2010. FIGURA Na �gura a seguir, as três circunferências em traçocontínuo são tangentes às retas r e s e a circunferência tracejada passa pelospontos A,B,C e D. Além disso, a circunferência menor é tangente tambéma AD e a circunferência maior é também tangente a BC. Se os raios dascircunferências externas ao quadrilátero ABCD são 8 e 18, calcule o raio Rda circunferência inscrita em ABCD.

2010. Uma mesa de bilhar tem o formato de um quadrado ABCD.SuperPablo tem uma missão especial: ele deve dar uma tacada em umabola de bilhar, inicialmente colocada no vértice A, de modo que, após baterexatamente 2010 vezes nos lados do quadrado, a bola chegue, pela primeiravez, a um vértice do quadrado. Quantos são os possíveis valores do ânguloformado pelo lado AB com a trajetória inicial da bola?

Observação: ao bater nos lados do quadrado, a bola sofre re�exão per-feita, ou seja, o ângulo de incidência é igual ao ângulo de re�exão. Suponhatambém que a bola seja um ponto.

2011. O ângulo interno do vértice A de um triângulo acutângulo ABCmede 75 graus. A altura relativa ao vértice A toca o lado BC no ponto D.As distâncias de D ao vértice B e ao ortocentro do triângulo são ambas iguaisa 10cm. Qual é a área do triângulo ABC, aproximada para o inteiro maispróximo? Se necessário, use

√3 ' 1, 732 .

2011. No triângulo ABC, o ângulo BC mede 45o. O círculo de diâmetroBC corta os lados AB e AC emD e E, respectivamente. Dado queDE = 10,encontre a distância do ponto médio M de BC à reta DE.

2012.Figura Uma tira retangular de papel ABCD é dobrada ao longodas linhas EF e HG de forma tal que os vértices A e B são levados para ummesmo ponto A? da mediatriz do segmento AB e o ângulo ∠HA?E é reto.Obtém-se assim o pentágono A?EFGH.

Sabe-se que as bordas inferiores da tira (segmentos FC? e GD? na �gura)se cortam no ponto médio M do lado AB. O lado menor da tira mede 1 e amedida do lado maior mede a+

√b , com a e b inteiros positivos. Quanto é

a+ b?

Page 81: Alan Anderson 30 de janeiro de 2017 · 6.4.10 Áreas para encontrar razões entre ... façam o mesmo com os outros níveis e ... Agora esse texto visa ajudar estudantes que não sabem

6.4. GEOMETRIA 81

2012. Dois círculos se cortam em dois pontos A e B. Seja X um pontosobre o segmento AB. Dez retas, todas passando por X, cortam os círculosem um total de quarenta pontos, quatro para cada reta. Qual é a quanti-dade mínima de quadriláteros cíclicos cujos quatro vértices estão entre essesquarenta pontos? Obs: um quadrilátero é cíclico se, e somente se, existe umcírculo que passa por seus quatro vértices.

2012. Sejam ABCD um quadrado, E o ponto médio do lado BC, Fo ponto médio do lado CD. Constroem-se os triângulos equiláteros ABG eBEH de forma que G está no interior do quadrado, e H no seu exterior.Determine o ângulo agudo entre as retas BF e GH.

2013. Pode-se provar que num triângulo acutângulo ABC, o triânguloDEF com D,E e F sobre os lados BC,CAe AB respectivamente com perí-metro mínimo é obtido quando D,E eF são as interseções das alturas comos lados. Tal triângulo é o triângulo órtico de ABC. Se AB = 13, BC = 14e CA = 15, o perímetro de seu triângulo órtico pode ser escrito na forma a

b,

com a e b inteiros primos entre si. Determine o valor de a+ b.

2014. Uma caixa de madeira em forma de paralelepípedo retângulo pos-sui dimensões 3 × 4 × 6. Ela está sobre o chão com uma de suas facescompletamente apoiada sobre o chão. Uma fontede luz emite raios paralelosde luz formando 45o com o chão. Considerando apenas essa fonte de luz,qual a área da maior sombra possível da caixa no chão? Não inclua a baseda caixa na sombra.

2015. Duas circunferências C1 e C2 se intersectam nos pontos A e B. Atangente a C1 por A corta C2 novamente no ponto P e a tangente a C2 porA corta C1 novamente no ponto Q. Sabendo que PB = 640 e QB = 1000,determine o comprimento do segmento AB.

2015. Seja ABCD um paralelogramo com AB = 8 e BC = 4. O círculoΓ passa por A,C e pelo ponto médio M de BC, e corta o lado CD no pontoP 6= C. Sabe-se que AD é tangente a Γ. Calcule a medida do segmentoMP .

Page 82: Alan Anderson 30 de janeiro de 2017 · 6.4.10 Áreas para encontrar razões entre ... façam o mesmo com os outros níveis e ... Agora esse texto visa ajudar estudantes que não sabem

82 CAPÍTULO 6. ENUNCIADOS DOS PROBLEMAS

6.4.2 A soma dos ângulos internos

2005.(�gura) Na �gura, ABCDE é um pentágono regular e AEF é umtriângulo eqüilátero. Seja P um ponto sobre o segmento BF , no interior deABCDE, e tal que o ângulo PEA mede 12o, como mostra a �gura abaixo.Calcule a medida, em graus, do ângulo PAC.

6.4.3 Semelhança de Triângulos e Relações Métricas emTriângulos Retângulos

1998. Sobre os lados AB e AC de um triângulo acutângulo ABC são cons-truídos, exteriormente ao triângulo, semicírculos tendo estes lados como di-âmetros. As retas contendo as alturas relativas aos lados AB e AC cortamesses semicírculos nos pontos P e Q. Prove que AP = AQ.

2004. Seja ABCD um trapézio retângulo de bases AB e CD, com ân-gulos retos em A e D. Dado que a diagonal menor BD é perpendicular aolado BC, determine o menor valor possível para a razão CD

AD.

2006. No triângulo ABC tem-se AB = 4, AC = 3 e o ângulo BAC mede60o. Seja D o ponto de intersecção entre a reta perpendicular a AB passandopor B e a reta perpendicular a AC passando por C. Determine a distânciaentre os ortocentros dos triângulos ABC e BCD.

2007. No quadrilátero convexo ABCD, ∠A+∠B = 120◦, AD = BC = 5e AB = 8. Externamente ao lado CD, construímos o triângulo eqüiláteroCDE. Calcule a área do triângulo ABE.

2008. Um trapézio isósceles ABCD, com lados paralelos AB e CD, étal que a diagonal BD mede 100m e o ângulo BDC mede 30◦. Seja S a áreado trapézio em m2. Determine S ·

√3.

2012. No triângulo ABC, seja AD a altura relativa a BC. Quantostriângulos não congruentes satisfazem 1

AB2 + 1AC2 = 1

AD2 com AD = 2012 eBD e CD ambos inteiros? Note que AB e AC não precisam ser inteiros.

Page 83: Alan Anderson 30 de janeiro de 2017 · 6.4.10 Áreas para encontrar razões entre ... façam o mesmo com os outros níveis e ... Agora esse texto visa ajudar estudantes que não sabem

6.4. GEOMETRIA 83

6.4.4 Lei dos Cossenos

2013. O quadrado ABCD está inscrito em um círculo de raio 30. A cordaAM corta a diagonal BD no ponto P . Se AM = 50, encontre o valor de AP .

2014. Um círculo tangencia os lados do quadrilátero ABCD . Os pon-tos de tangência são R sobre AB, S sobre BC,T sobre CD e U sobre DA.Sabe-se que AU = 1,DU = 2, BS = 2 e CS = 4 . Calcule o comprimento SU .

6.4.5 Lei dos senos

2000. O trapézio ABCD tem bases AB e CD. O lado DA mede x e o ladoBC mede 2x. A soma dos ângulos DAB e ABC é 120o. Determine o ânguloDAB.

2001. No triângulo ABC, a mediana e a altura relativas ao vértice Adividem o ângulo BC em três ângulos de mesma medida. Determine as me-didas dos ângulos do triângulo ABC.

2010. As bissetrizes internas dos ângulos A e C do triângulo ABCcortam-se no ponto I. Sabe-se que AI = BC e que m(ICA) = 2m(IAC).Determine a medida do ângulo ABC.

6.4.6 Desigualdade Triangular

2003. Entre 15 números reais distintos, o menor deles igual a 1, não há trêsque podem ser lados de um triângulo. Quais valores o maior dos 15 númerospode assumir?

6.4.7 Área do Triângulo

2005.(�gura) Um terreno quadrangular foi dividido em quatro lotes meno-res por duas cercas retas unindo os pontos médios dos lados do terreno. Asáreas de três dos lotes estão indicadas em metros quadrados no mapa ao lado.

Page 84: Alan Anderson 30 de janeiro de 2017 · 6.4.10 Áreas para encontrar razões entre ... façam o mesmo com os outros níveis e ... Agora esse texto visa ajudar estudantes que não sabem

84 CAPÍTULO 6. ENUNCIADOS DOS PROBLEMAS

Qual é a área do quarto lote, representado pela região destacada no mapa?2009. No triângulo retângulo ABC, ∠A = 90o, AB = 5cm e BC = 9cm.

Se I é o incentro de ABC, determine o comprimento do segmento CI.

6.4.8 Pontos notáveis

1999. Seja ABCD um quadrado. Escolhemos pontos M,N,P,Q respecti-vamente sobre AB,BC,CD e DA, de modo que as circunferências circuns-critas aos triângulos MBN e PDQ sejam tangentes exteriormente. Mostreque MN + PQ ≥ AC.

2002. Em um quadrilátero convexo ABCD, os lados opostos AD e BCsão congruentes e os pontos médios das diagonais AC e BD são distintos.Prove que a reta determinada pelos pontos médios das diagonais forma ân-gulos iguais com AD e BC.

2004. Sejam H, I e O o ortocentro, o incentro e o circuncentro do tri-ângulo ABC, respectivamente. A reta CI corta o circuncírculo de ABC noponto L, distinto de C. Sabe-se que AB = IL e AH = OH. Determine osângulos do triângulo ABC.

2007. O triângulo ABC é retângulo em B. Sejam I o centro da circun-ferência inscrita em ABC e O o ponto médio do lado AC. Se AOI = 45o,quanto mede, em graus, o ângulo ACB?

6.4.9 Potência de Ponto

2015. Seja ABCD um paralelogramo com AB = 8 e BC = 4. O círculo Γpassa por A, C e pelo ponto médio M de BC, e corta o lado CD no pontoP 6= C. Sabe-se que AD é tangente a Γ. Calcule a medida do segmentoMP .

6.4.10 Áreas para encontrar razões entre segmentos

2014. Figura No desenho abaixo, o triângulo ABC é equilátero e BD =CE = AB/3 . A razão EG/GD pode ser escrita na formam/n, mdc(m,n) =

Page 85: Alan Anderson 30 de janeiro de 2017 · 6.4.10 Áreas para encontrar razões entre ... façam o mesmo com os outros níveis e ... Agora esse texto visa ajudar estudantes que não sabem

6.4. GEOMETRIA 85

1. Quanto vale m+ n?

Page 86: Alan Anderson 30 de janeiro de 2017 · 6.4.10 Áreas para encontrar razões entre ... façam o mesmo com os outros níveis e ... Agora esse texto visa ajudar estudantes que não sabem

86 CAPÍTULO 6. ENUNCIADOS DOS PROBLEMAS

Page 87: Alan Anderson 30 de janeiro de 2017 · 6.4.10 Áreas para encontrar razões entre ... façam o mesmo com os outros níveis e ... Agora esse texto visa ajudar estudantes que não sabem

Capítulo 7

Ideias e Soluções

7.1 Soluções Álgebra

7.1.1 Sequências e funções

2015. A solução abaixo não é a do problema, em vez disso iremos mostrarcomo contar o número de x tais que g(x) = 0.

Observe que se x > 1 é ímpar, então podemos escrever x = 2k + 1, comk > 0 e g(x) = g(2k + 1) = k > 0. Portanto, se g(x) = 0, temos que xé par. Agora se x > 0 é um número par, podemos escrever x = 2x1 comx1 > 0. Daí temos que g(2x1) = f(x1). Agora, se x1 é par, então podemosescrever x1 = 2x2, com x2 > 0 e g(x) = f(x1) = f(2x2) = x2 > 0. Portanto,se x é da forma 4x2, para algum x2 > 0, então g(x) > 0. Assim, para queg(x) = 0 devemos ter que x seja da forma 4k + 2 (pois nem pode ser ímpar,nem da forma 4k). Agora suponho que x = 4k+ 2, então g(x) = g(4k+ 2) =f(2k + 1) = g(k).

Vamos analizar o número na base 4. Observe que se k = (aj...a0)4 ex = 4k + 2, então x = (aj...a02)4. Pelo que vimos, para que g(x) = 0devemos ter que x seja da forma 4k + 2, e assim x tem que terminar como dígito 2 quando escrito na base 4. Como g(x) = g(k), para que g(x) = 0devemos ter g(k) = 0, logo o último de dígito de k deve ser 2 também.Assim, repetindo esse processo, todos os dígitos diferentes do primeiro, ouseja, a0, a1, ..., aj−1 devem ser iguais a 2 e além disso g(aj) deve ser igual azero, logo aj = 1 ou aj = 2. Portanto os números tais que g(x) = 0 são osnúmeros das formas (122...22)4 ou (222...22)4. Basta veri�car qual é o maior

87

Page 88: Alan Anderson 30 de janeiro de 2017 · 6.4.10 Áreas para encontrar razões entre ... façam o mesmo com os outros níveis e ... Agora esse texto visa ajudar estudantes que não sabem

88 CAPÍTULO 7. IDEIAS E SOLUÇÕES

número em cada uma dessas formas e encontramos facilmente o resultado.